You are on page 1of 647

Intro Neurology

Intro Neurology
In this video…
• 200 Neurology review
questions
• Fun mnemonics!!
• Updates in neurology
Q201 Neurology
The best initial step in suspected stroke
assessment is:

A) MRI
B) CT without contrast
C) CT with contrast
Q201 Neurology
The best initial step in suspected stroke
assessment is:

A) MRI
B) CT without contrast
C) CT with contrast
Q201 Neurology
The best initial step in suspected stroke
assessment is: CT without contrast helps
differentiate ischemic (85% of time)
A) MRI from hemorrhagic (15%) strokes
B) CT without contrast
C) CT with contrast
Q201 Neurology
The best initial step in suspected stroke
assessment is: CT without contrast helps
differentiate ischemic (85% of time)
A) MRI from hemorrhagic (15%) strokes
B) CT without contrast NO treatment for stroke without
doing the CT w/o contrast!!
first
C) CT with contrast
Q201 Neurology
The best initial step in suspected stroke
assessment is: CT without contrast helps
differentiate ischemic (85% of time)
A) MRI from hemorrhagic (15%) strokes
B) CT without contrast NO treatment for stroke without
doing the CT w/o contrast!!
first
C) CT with contrast
Q201 Neurology
The best initial step in suspected stroke
assessment is: CT without contrast helps
differentiate ischemic (85% of time)
A) MRI from hemorrhagic (15%) strokes
B) CT without contrast NO treatment for stroke without
doing the CT w/o contrast!!
first
C) CT with contrast
Remember! Kidney stone
evaluation should also be
done using CT w/o contrast!
Q202 Neurology
What is the most common and most important risk
factor for stroke? The most common location of atherosclerotic
plaque buildup is the carotid bifurcation

A) CAD
B) Obesity
C) Diabetes
D) Afib
E) Drug use
F) Hypertension
Q202 Neurology
What is the most common and most important risk
factor for stroke? The most common location of atherosclerotic
plaque buildup is the carotid bifurcation

A) CAD
B) Obesity
C) Diabetes
D) Afib
E) Drug use
F) Hypertension
Q202 Neurology
What is the most common and most important risk
factor for stroke? The most common location of atherosclerotic
plaque buildup is the carotid bifurcation

Age is also a big risk factor,


A) CAD (non-modifiable); and:
• smoking
B) Obesity • high cholesterol
• carotid stenosis
C) Diabetes• family history of MI/stroke,
D) Afib • age >60
• male gender
E) Drug use• black, Hispanic, Asian

F) Hypertension
Q203 Neurology
What is the first thing to do for a patient with
ischemic stroke (<4.5 hours of symptoms)?

A) Aspirin
B) Alteplase
Q203 Neurology
What is the first thing to do for a patient with
ischemic stroke (<4.5 hours of symptoms)?

A) Aspirin
B) Alteplase
Q203 Neurology
What is the first thing to do for a patient with
ischemic stroke (<4.5 hours of symptoms)?
IMPORTANT POINTS
A) Aspirin • tPa (given IV, breaks up clots) – DO
NOT GIVE in hemorrhagic stroke!!! It
B) Alteplase will worsen bleeding
• tPa must be given within 4.5 hours
• Aspirin/clopidogrel (dual antiplatelet
therapy) are given 24 hours after tPA
for about 21 days
• Fever and hyperglycemia (esp. > 400)
are associated with poor outcomes in
acute stroke – treat them!
Q203 Neurology
What is the first thing to do for a patient with
ischemic stroke (<4.5 hours of symptoms)?
Contraindications to tPA include:
A) Aspirin -stroke or head trauma within last 3 months
-major surgery in last 2 weeks
B) Alteplase -glucose less than 50
-prior IC hemorrhage
-GI or urinary bleed in the past 21 days
-systolic > 185, diastolic > 110 (try to correct [e.g.,
with labetalol] within 3 hours to be able to give tPa)
Q203 Neurology
What is the first thing to do for a patient with
ischemic stroke (<4.5 hours of symptoms)?
Contraindications to tPA include:
A) Aspirin -stroke or head trauma within last 3 months
-major surgery in last 2 weeks
B) Alteplase -glucose less than 50
-prior IC hemorrhage
-GI or urinary bleed in the past 21 days
-systolic > 185, diastolic > 110 (try to correct [e.g.,
with labetalol] within 3 hours to be able to give tPa)
A patient is brought in with ischemic stroke and it’s been
10 hours since onset – what should you do?!?
Q203 Neurology
What is the first thing to do for a patient with
ischemic stroke (<4.5 hours of symptoms)?
Contraindications to tPA include:
A) Aspirin -stroke or head trauma within last 3 months
-major surgery in last 2 weeks
B) Alteplase -glucose less than 50
-prior IC hemorrhage
-GI or urinary bleed in the past 21 days
-systolic > 185, diastolic > 110 (try to correct [e.g.,
with labetalol] within 3 hours to be able to give tPa)
A patient is brought in with ischemic stroke and it’s been
10 hours since onset – what should you do?!?
Thrombectomy!!
Q203 Neurology
What is the first thing to do for a patient with
ischemic stroke (<4.5 hours of symptoms)?
Contraindications to tPA include:
A) Aspirin -stroke or head trauma within last 3 months
-major surgery in last 2 weeks
B) Alteplase -glucose less than 50
-prior IC hemorrhage
-GI or urinary bleed in the past 21 days
-systolic > 185, diastolic > 110 (try to correct [e.g.,
with labetalol] within 3 hours to be able to give tPa)
A patient is brought in with ischemic stroke and it’s been
10 hours since onset – what should you do?!?
Thrombectomy!! can be done for 24 hours
Q204 Neurology
Carotid endarterectomy in a currently
asymptomatic patient is indicated:

A) never
B) if there is 80% stenosis
Q204 Neurology
Carotid endarterectomy in a currently
asymptomatic patient is indicated:

A) never
B) if there is 80% stenosis
(some say 70%)
Q204 Neurology
Carotid endarterectomy in a currently
asymptomatic patient is indicated:
if >70-80% stenosis, endarterectomy!
if 50-70%, unclear data (won’t be tested)! (maybe if symptomatic)
A) never if <50%, NO benefit
B) if there is 80% stenosis
Q204 Neurology
Carotid endarterectomy in a currently
asymptomatic patient is indicated:
if >70-80% stenosis, endarterectomy!
if 50-70%, unclear data (won’t be tested)! (maybe if symptomatic)
A) never if <50%, NO benefit
B) if there is 80% stenosis
Q204 Neurology

Long term management after TIA/stroke:


-carotid endarterectomy (if indicated) [better than
angioplasty]; get echo to look for afib
-hypertension management (diastolic below 80)
-diabetes management (blood sugar at about 100)
-statin for blood lipids; diet, exercise (LDL to >70)
-dual antiplatelets to prevent clot formation
-for cardioembolic strokes, anticoagulation
Q205 Neurology
A 65-year-old man has a transient episode of
weakness in his right hand and blurred vision,
without associated headache. What occurred?

A) Left sided carotid TIA


B) Left MCA stroke
Q205 Neurology
A 65-year-old man has a transient episode of
weakness in his right hand and blurred vision,
without associated headache. What occurred?

A) Left sided carotid TIA


B) Left MCA stroke
Q205 Neurology
A 65-year-old man has a transient episode of
weakness in his right hand and blurred vision,
without associated headache. What occurred?

A) Left sided carotid TIA


B) Left MCA stroke
A TIA is an event in which symptoms
last less than 24 hours! (although
usually less than 1 hour)
Q205 Neurology
A 65-year-old man has a transient episode of
weakness in his right hand and blurred vision,
without associated headache. What occurred?
And remember! Amaurosis fugax is
A) Left sided carotid TIA temporary loss of vision in one or both

B) Left MCA stroke


eyes due to lack of blood flow to the retina
seen sometimes in TIA's; vision is
described as appearing as a "black curtain
coming down vertically into the field of
vision in one eye".
[TIA’s are treated with antiplatelets!!!]
Q206 Neurology
A man experiences a stroke and now has trouble
formulating the words he wants to say. What lobe
was affected?

A) Temporal
B) Frontal
Q206 Neurology
A man experiences a stroke and now has trouble
formulating the words he wants to say. What lobe
was affected?

A) Temporal
B) Frontal
Q206 Neurology
A man experiences a stroke and now has trouble
formulating the words he wants to say. What lobe
was affected?

A) Temporal
B) Frontal
95% of people are
left dominant,
Broca’s is in the
LEFT frontal lobe
Q207 Neurology
In Brown Sequard syndrome, there is contralateral
deficits in:

A) Motor function
B) Proprioception
C) Pain/temp sensation
Q207 Neurology
In Brown Sequard syndrome, there is contralateral
deficits in:

A) Motor function
B) Proprioception
C) Pain/temp sensation
Q207 Neurology
In Brown Sequard syndrome, there is contralateral
deficits in:

A) Motor function
B) Proprioception
C) Pain/temp sensation
• In Brown Sequard Syndrome, pain and
temperature sensation (3) are lost a few segments
below the level of the lesion [spinothalamic tract]
• There will also be ipsilateral motor and
proprioception deficits (1 and 2)
Q208 Neurology
Hyperextension injury (e.g., cervical spine) is most
associated with :

A) Central cord syndrome


B) Anterior cord syndrome
C) Brown-Sequard syndrome
Q208 Neurology
Hyperextension injury (e.g., cervical spine) is most
associated with :

A) Central cord syndrome


B) Anterior cord syndrome
C) Brown-Sequard syndrome
Q208 Neurology
Hyperextension injury (e.g., cervical spine) is most
associated with :

A) Central cord syndrome – UE worse than LE


B) Anterior cord syndrome – proprioception intact
C) Brown-Sequard syndrome - hemisection
Q209 Neurology
If the purple areas represent demyelination, what
condition is this?

A) Polio
B) MS
C) Tabes Dorsalis
D) Cobalamin deficiency
Q209 Neurology
If the purple areas represent demyelination, what
condition is this?

A) Polio
B) MS
C) Tabes Dorsalis
D) Cobalamin deficiency
Q209 Neurology
If the purple areas represent demyelination, what
condition is this?

A) Polio affects anterior horns; LMN lesions only; flaccid paralysis

B) MS demyelination; random lesions of white matter, mostly cervical

C) Tabes Dorsalis 3o syphilis; impaired proprioception; +Romberg sign

D) Cobalamin deficiency – subacute combined degeneration


Q210 Neurology
The biceps reflex is mediated by:

A) S1-S2
B) L3-L4
C) C5-C6
D) C7-C8
Q210 Neurology
The biceps reflex is mediated by:

A) S1-S2
B) L3-L4
C) C5-C6
D) C7-C8
Q210 Neurology
The biceps reflex is mediated by:

A) S1-S2 - tie your shoe (Achilles)


B) L3-L4 – kick the floor (Patella)
C) C5-C6 – pick up sticks (Biceps)
D) C7-C8 – lay them straight (Triceps)
and L1-L2, the testis move (cremasteric reflex)
Q211 Neurology
Hip flexion is controlled by
which nerve?

A) Femoral nerve
B) Obturator nerve
C) Sciatic nerve
Q211 Neurology
Hip flexion is controlled by
which nerve?

A) Femoral nerve
B) Obturator nerve
C) Sciatic nerve
Q211 Neurology
Hip flexion is controlled by
which nerve?

A) Femoral nerve
B) Obturator nerve
C) Sciatic nerve
Femoral nerve – innervates anterior
thigh muscles and controls knee
extension and hip flexion
Q212 Neurology
First-line for trigeminal neuralgia is:

A) Carbamazepine
B) Baclofen
Q212 Neurology
First-line for trigeminal neuralgia is:

A) Carbamazepine
B) Baclofen
Q212 Neurology
First-line for trigeminal neuralgia is:

A) Carbamazepine
B) Baclofen – may be
given if carbamazepine
is contraindicated
(e.g., bone marrow
suppression)
Q212 Neurology
First-line for trigeminal neuralgia is:

A) Carbamazepine
B) Baclofen – may be
given if carbamzapine
is contraindicated
(e.g., bone marrow
suppression)
Baclofen (and tizanidine, an alpha-2 agonist) are the
most used medications for reducing spasticity in MS.
Q213 Neurology
Asymmetric muscle weakness is seen in:

A) Conus medullaris syndrome


B) Cauda equina syndrome
Q213 Neurology
Asymmetric muscle weakness is seen in:

A) Conus medullaris syndrome


B) Cauda equina syndrome
Q213 Neurology
Asymmetric muscle weakness is seen in:

A) Conus medullaris syndrome - symmetric


B) Cauda equina syndrome – asymmetric
Cauda equina syndrome
-due to disc herniation or
tumor; severe radicular Both Conus medullaris
pain require syndrome
-absent knee/ankle reflex, -often more abrupt onset
saddle anesthesia surgical -knee reflex intact (may
-muscle weakness on one evaluation! see hyperreflexia)
side -numbness typically only
-loss of anal sphincter in perianal area
control/urination -muscles affected
symmetrically
Q214 Neurology
If subarachnoid hemorrhage is suspected,
but CT is unrevealing, what is the next
best step?

A) Lumbar puncture
B) MR angiography
Q214 Neurology
If subarachnoid hemorrhage is suspected,
but CT is unrevealing, what is the next
best step?

A) Lumbar puncture
B) MR angiography
Q214 Neurology
If subarachnoid hemorrhage is suspected,
but CT is unrevealing, what is the next
best step?
To look for RBCs, xanthochromia (yellow CSF caused by RBC
breakdown), protein (from RBC’s), and increased ICP

A) Lumbar puncture
B) MR angiography
This is done once SAH has been confirmed
to identify the source of bleeding
Q214 Neurology
If subarachnoid hemorrhage is suspected,
but CT is unrevealing, what is the next
best step?
To look for RBCs, xanthochromia (yellow CSF caused by RBC
breakdown), protein (from RBC’s), and increased ICP

A) Lumbar puncture
B) MR angiography
This is done once SAH has been confirmed
to identify the source of bleeding
SAH symptoms: e.g., “worst headache of my life”, sudden onset,
neck stiffness (meningeal irritation), photopia, vomiting
Q214 Neurology
If subarachnoid hemorrhage is suspected,
but CT is unrevealing, what is the next
best step?
To look for RBCs, xanthochromia (yellow CSF caused by RBC
breakdown), protein (from RBC’s), and increased ICP

A) Lumbar puncture
B) MR angiography
This is done once SAH has been confirmed
to identify the source of bleeding
Keep BP down to prevent rebleeding (most likely to occur in first 24 hours)
Give nimodipine to prevent vasospasm and subsequent ischemic stroke (days 4-10 post SAH)
Q214 Neurology
If subarachnoid hemorrhage is suspected,
but CT is unrevealing, what is the next
best step?
To look for RBCs, xanthochromia (yellow CSF caused by RBC
breakdown), protein (from RBC’s), and increased ICP

A) Lumbar puncture NOTE: Blood in CSF in

B) MR angiography
patient with big headache?
Most likely SAH!
This is done once SAH has been confirmed
to identify the source of bleeding
Keep BP down to prevent rebleeding (most likely to occur in first 24 hours)
Give nimodipine to prevent vasospasm and subsequent ischemic stroke (days 4-10 post SAH)
Q215 Neurology
A man with PKD is admitted for SAH. Four
hours after admission he develops
weakness in his left arm. What is the
cause?

A) Rebleeding
B) Vasospasm
Q215 Neurology
A man with PKD is admitted for SAH. Four
hours after admission he develops
weakness in his left arm. What is the
cause?

A) Rebleeding
B) Vasospasm
Q215 Neurology
A man with PKD is admitted for SAH. Four
hours after admission he develops
weakness in his left arm. What is the
cause?

A) Rebleeding – w/ in 24 hours
B) Vasospasm – after 4 days
Q215 Neurology
A man with PKD is admitted for SAH. Four
hours after admission he develops
weakness in his left arm. What is the
cause?

A) Rebleeding – w/ in 24 hours
B) Vasospasm – after 4 days
Conditions associated with berry aneurysms that can MAKE SAH more likely:
-Marfan, Aortic coarctation, Kidney disease (PKD), Ehlers-Danlos, Sickle
cell/Smoking, Atherosclerosis, Hypertension
Q216 Neurology
A 50-year-old presents with a two-hour
history of numbness and droop on the
right side of his face, difficulty talking,
and weakness in his right arm. Where is
the lesion?

A) ACA
B) MCA
Q216 Neurology
A 50-year-old presents with a two-hour
history of numbness and droop on the
right side of his face, difficulty talking,
and weakness in his right arm. Where is
the lesion?

A) ACA
B) MCA
Q216 Neurology
A 50-year-old presents with a two-hour
history of numbness and droop on the
right side of his face, difficulty talking,
and weakness in his right arm. Where is
the lesion?
The lesion is in the left MCA; that explains
the contralateral facial and UE weakness;
A) ACA in addition, this explains the aphasia as

B) MCA
Broca's area is affected;
Next step? CT w/o contrast!! To rule out
hemorrhage and perhaps begin tPA!!!
Q217 Neurology
Which of the following patients would
be most likely to present with a
chronic subdural hematoma?

A) Woman with cerebral aneurysm


B) 72-year-old male alcoholic
Q217 Neurology
Which of the following patients would
be most likely to present with a
chronic subdural hematoma?

A) Woman with cerebral aneurysm


B) 72-year-old male alcoholic
Q217 Neurology
Which of the following patients would
be most likely to present with a
chronic subdural hematoma?

A) Woman with cerebral aneurysm


B) 72-year-old male alcoholic
Elderly + chronic alcohol = chronic
asymptomatic subdural hematoma. Patients
often have brain atrophy and more fragile veins
that are more likely to tear.
Q217 Neurology
Which of the following patients would
be most likely to present with a
chronic subdural hematoma?

A) Woman with cerebral aneurysm


B) 72-year-old male alcoholic
Subdural hematoma occurs acutely (major trauma) or chronically;
may cause fatigue, nausea, dizziness, confusion, and seizures; CT
findings are “banana” shaped; isodense subacutely, hypodense
chronically; treat with surgery if symptomatic (may regress
spontaneously)
Q218 Neurology
A 24-year-old female presents with worsening
periorbital headache, fever, and double vision. (She
attempted to squeeze a mid-facial furuncle a week
ago.) PE shows bilateral supraorbital edema and
lateral gaze palsy. What is the most likely
organism?

A) Staph
B) Strep
Q218 Neurology
A 24-year-old female presents with worsening
periorbital headache, fever, and double vision. (She
attempted to squeeze a mid-facial furuncle a week
ago.) PE shows bilateral supraorbital edema and
lateral gaze palsy. What is the most likely
organism?

A) Staph
B) Strep
Q218 Neurology
A 24-year-old female presents with worsening
periorbital headache, fever, and double vision. (She
attempted to squeeze a mid-facial furuncle a week
ago.) PE shows bilateral supraorbital edema and
lateral gaze palsy. What is the most likely
organism? infection (commonly from skin, orbit) that leads to
Cavernous sinus thrombosis - uncontrolled

septic thrombosis of the cavernous sinus (70%


staph aureus). Headache, orbital pain, edema,
A) Staph vision. Treat with empiric antibiotics (vanco,
ceftriaxone, and metronidazole; steroids; consider
B) Strep surgical drainage if no response). 30% mortality
Q219 Neurology
How is migraine diagnosed?

A) MRI
B) Patient history
Q219 Neurology
How is migraine diagnosed?

A) MRI
B) Patient history
Q219 Neurology
How is migraine diagnosed?

A) MRI
B) Patient history negative workup; MRI doesn’t show anything
Migraine diagnosis is based on history and an otherwise
Q219 Neurology
How is migraine diagnosed?

A) MRI
B) Patient history negative workup; MRI doesn’t show anything
Migraine diagnosis is based on history and an otherwise

MRI treatment:
-avoid known triggers, sleep hygiene
-abort: NSAIDs, triptans, [ergotamine]
-prophylaxis: propranolol (esp. in pregnancy), CGRP’s (the
“–mabs”), anticonvulsants (e.g., valproate, gabapentin),
TCA’s (e.g., amitriptyline)
Q220 Neurology
What is first-line prophylactic for cluster
headaches?

A) Sumatriptan
B) Verapamil
Q220 Neurology
What is first-line prophylactic for cluster
headaches?

A) Sumatriptan
B) Verapamil
Q220 Neurology
What is first-line prophylactic for cluster
headaches?
sumatriptan is for abortive therapy (not really used); high flow O2
A) Sumatriptan
B) Verapamil
Verapamil is first line for
prophylactic;
alternatives include
lithium, valproic acid,
prednisone, and
topiramate
Q221 Neurology
A 53-year-old mother presents with new-onset
headaches similar in presentation to her
daughter’s cluster headaches. Next step?

A) MRI
B) get ESR
Q221 Neurology
A 53-year-old mother presents with new-onset
headaches similar in presentation to her
daughter’s cluster headaches. Next step?

A) MRI
B) get ESR
Q221 Neurology
A 53-year-old mother presents with new-onset
headaches similar in presentation to her
daughter’s cluster headaches. Next step?

A) MRI
B) get ESR
New onset unilateral headache in
patients above 50 is concerning for
giant cell arteritis; get ESR, even if
headaches are mild with no
associated symptoms
Q222 Neurology
A 76-year-old woman has afib. She has no
significant medical history other than controlled
hypertension. What is her CHA2DS2VASc score and
what should be given?

A) 1, aspirin
B) 2, warfarin
C) 3, warfarin
D) 4, DOAC dabigatran)
(e.g., apixaban,
Q222 Neurology
A 76-year-old woman has afib. She has no
significant medical history other than controlled
hypertension. What is her CHA2DS2VASc score and
what should be given?

A) 1, aspirin
B) 2, warfarin
C) 3, warfarin
D) 4, DOAC dabigatran)
(e.g., apixaban,
Q222 Neurology
A 76-year-old woman has afib. She has no
significant medical history other than controlled
hypertension. What is her CHA2DS2VASc score and
what should be given?
She is 76: 2 points.
She’s female: 1 point.
A) 1, aspirin HTN: 1 point.

B) 2, warfarin If 1 point, give aspirin;


C) 3, warfarin (if not, then give
if 2 points give DOAC

D) 4, DOAC warfarin)
Q223 Neurology
A child experiences multiple intermittent 5-second
episodes staring into space. What is the
treatment?

A) Lorazepam
B) Ethosuximide
Q223 Neurology
A child experiences multiple intermittent 5-second
episodes staring into space. What is the
treatment?

A) Lorazepam
B) Ethosuximide
Q223 Neurology
A child experiences multiple intermittent 5-second
episodes staring into space. What is the
treatment?

A) Lorazepam
B) Ethosuximide
Most patients can
discontinue medication
before puberty with no long-
term sequelae.
Second line: valproic acid
Q223 Neurology
A child experiences multiple intermittent 5-second
episodes staring into space. What is the
treatment? Childhood absence epilepsy
-form of generalized seizure
-presents with 5-10 second periods of
A) Lorazepam impaired consciousness
-can occur more than 30 times a day
B) Ethosuximide -looks like child is daydreaming/staring
Most patients can -no postictal phase
discontinue medication -can be triggered by hyperventilation
before puberty with no long- -subsides before adulthood
term sequelae. -diagnose with EEG! shows generalized 3-
Second line: valproic acid Hz spike-wave activity
Q224 Neurology
A 39-year-old man presents with a single simple
partial seizure of 1 minute but is no longer
symptomatic. He also complains of recent morning
headaches and one episode of vomiting. Next step?

A) Empiric antibiotics
B) CT scan
Q224 Neurology
A 39-year-old man presents with a single simple
partial seizure of 1 minute but is no longer
symptomatic. He also complains of recent morning
headaches and one episode of vomiting. Next step?

A) Empiric antibiotics
B) CT scan
Q224 Neurology
A 39-year-old man presents with a single simple
partial seizure of 1 minute but is no longer
symptomatic. He also complains of recent morning
headaches and one episode of vomiting. Next step?

A) Empiric antibiotics
B) CT scan History is suggestive of a brain tumor;
if seizures recur, consider
anticonvulsant therapy
Q225 Neurology
A man is brought in with pure motor hemiparesis of
the left face, arm, and leg. Nearby is a woman who
presents with pure left sided sensory deficits of
the face, arm and leg. What do they both have?

A) Wallenberg syndrome
B) Lacunar strokes
Q225 Neurology
A man is brought in with pure motor hemiparesis of
the left face, arm, and leg. Nearby is a woman who
presents with pure left sided sensory deficits of
the face, arm and leg. What do they both have?

A) Wallenberg syndrome
B) Lacunar strokes
Q225 Neurology
A man is brought in with pure motor hemiparesis of
the left face, arm, and leg. Nearby is a woman who
presents with pure left sided sensory deficits of
the face, arm and leg. What do they both have?
Hypertension & atherosclerosis
are highly associated w/ lacunar
A) Wallenberg syndrome strokes; **pure sensory stroke
B) Lacunar strokes (i.e., at VPL of thalamus)** or
**pure motor (i.e., at internal
capsule)**; other types not
really tested
Q225 Neurology
A man is brought in with pure motor hemiparesis of
the left face, arm, and leg. Nearby is a woman who
presents with pure left sided sensory deficits of
the face, arm and leg. What do they both have?
Loss of pain and temperature sensation on ipsilateral face and Hypertension & atherosclerosis
contralateral body; ipsilateral bulbar weakness, vertigo, nystagmus
are highly associated w/ lacunar
A) Wallenberg syndrome strokes; **pure sensory stroke
B) Lacunar strokes (i.e., at VPL of thalamus)** or
**pure motor (i.e., at internal
capsule)**; other types not
really tested
Q226 Neurology
What is the correct management for hypertensive
emergency?

A) Beta blockers, reduce pressure over 24 hours


B) Nitroprusside, reduce immediately
Q226 Neurology
What is the correct management for hypertensive
emergency?

A) Beta blockers, reduce pressure over 24 hours


B) Nitroprusside, reduce immediately
Q226 Neurology
What is the correct management for hypertensive
emergency?
Management for hypertensive urgency (elevated BP with moderate symptoms, e.g., headache, w/o end
organ damage) is reduction of BP over 24-48 hours; e.g., beta blockers, ACEi’s
A) Beta blockers, reduce pressure over 24 hours
B) Nitroprusside, reduce immediately
Also: labetalol, nicardipine, hydralazine
Remember, hypertensive emergency means:
Elevated blood pressure with signs of end-organ
damage, such as AKI, retinal hemorrhage,
papilledema, stroke, MI, or pulmonary edema
Q227 Neurology
Tourette syndrome is diagnosed by motor tics (e.g.,
blinking) AND phonic tics (e.g., throat clearing)
every day or nearly every day for:

A) 1 month
B) 1 year
Q227 Neurology
Tourette syndrome is diagnosed by motor tics (e.g.,
blinking) AND phonic tics (e.g., throat clearing)
every day or nearly every day for:

A) 1 month
B) 1 year
Q227 Neurology
Tourette syndrome is diagnosed by motor tics (e.g.,
blinking) AND phonic tics (e.g., throat clearing)
every day or nearly every day for:
VMAT2 (tetrabenazine) inhibitors
are now preferred for Tourette's as

A) 1 month
initial treatment (over
antipsychotics) due to better effect
profile.

B) 1 year
If behavior therapy doesn’t work, medications
include:
-VMAT2 inhibitors, (haloperidol, risperidone)
-clonidine, guanfacine (less effective at tic
reduction, but less side effects; good for patient
with ADHD!)
Q228 Neurology
Patient complains of “room spinning”. Patient
is asked to turn her head 45 degrees right or
left and is then helped from a sitting to supine
position. Vertigo and nystagmus are
reproduced. What is the diagnosis?

A) Acute peripheral vestibulopathy


B) Benign paroxysmal positional vertigo
Q228 Neurology
Patient complains of “room spinning”. Patient
is asked to turn her head 45 degrees right or
left and is then helped from a sitting to supine
position. Vertigo and nystagmus are
reproduced. What is the diagnosis?

A) Acute peripheral vestibulopathy


B) Benign paroxysmal positional vertigo
Q228 Neurology
Benign paroxysmal positional vertigo

-due to an otolith that leads to disturbances in the semicircular


canals; does NOT cause hearing loss!!!
-presents with transient episodic vertigo (lasting <1 minute)
and nystagmus triggered by changes in head position
-diagnosis: Dix-Hallpike maneuver
-Tx: Epley maneuver (extended Dix Hallpike maneuver –
resolves 80% of cases), usually subsides in weeks to month;
but 30% recur within 1 year; anti-vertigo medicine (e.g.,
meclizine) are contraindicated!
Q229 Neurology
A man has 2 distinct episodes of vertigo (lasting >20
minutes), one episode of hearing loss, and tinnitus; what
is the treatment?

A) Meclizine
B) Diazepam
C) Promethazine
D) Salt restricted diet
E) All of the above
Q229 Neurology
A man has 2 distinct episodes of vertigo (lasting >20
minutes), one episode of hearing loss, and tinnitus; what
is the treatment?

A) Meclizine
B) Diazepam
C) Promethazine
D) Salt restricted diet
E) All of the above
Q229 Neurology
A man has 2 distinct episodes of vertigo (lasting >20
minutes), one episode of hearing loss, and tinnitus; what
is the treatment? Meniere disease – triad of vertigo, hearing loss, tinnitus

A) Meclizine
B) Diazepam
C) Promethazine
D) Salt restricted diet
E) All of the above
Q229 Neurology
A man has 2 distinct episodes of vertigo (lasting >20
minutes), one episode of hearing loss, and tinnitus; what
is the treatment? Meniere disease – triad of vertigo, hearing loss, tinnitus

A) Meclizine – controls acute vertigo


B) Diazepam - controls acute vertigo
C) Promethazine – anti-nausea/vomiting
D) Salt restricted diet – long term prevention
E) All of the above
Q229 Neurology
A man has 2 distinct episodes of vertigo (lasting >20
minutes), one episode of hearing loss, and tinnitus; what
is the treatment? Meniere disease – triad of vertigo, hearing loss, tinnitus
vertigo and vomiting a
week after being diagnosed
A) Meclizine – controls acute vertigo with a viral infection - it is

B) Diazepam - controls acute vertigo


likely acute vestibular
neuritis; self-resolving,

C) Promethazine – anti-nausea/vomiting
steroids may help

D) Salt restricted diet – long term prevention


E) All of the above Diuretics may also help
Q230 Neurology
A 66-year-old presents with a decade of bilateral hand
tremors (seen also in his mother and older brother).
Tremors are worse with movement. He denies difficulty
concentrating or falls. What is the treatment?

A) Caffeine
B) Propranolol
Q230 Neurology
A 66-year-old presents with a decade of bilateral hand
tremors (seen also in his mother and older brother).
Tremors are worse with movement. He denies difficulty
concentrating or falls. What is the treatment?

A) Caffeine
B) Propranolol
Q230 Neurology
A 66-year-old presents with a decade of bilateral hand
tremors (seen also in his mother and older brother).
Tremors are worse with movement. He denies difficulty
concentrating or falls. What is the treatment?
(could make it worse)

A) Caffeine
B) Propranolol
also, primidone (an anti-convulsant)
-if not, AEDs
Q231 Neurology
A woman with fluctuating ptosis and muscle weakness
has a positive “ice pack” test; labs show +acetylcholine
receptor antibody. What medications are
contraindicated?

A) Aminoglycosides
B) Fluoroquinolones
C) Magnesium sulfate
D) All of the above
Q231 Neurology
A woman with fluctuating ptosis and muscle weakness
has a positive “ice pack” test; labs show +acetylcholine
receptor antibody. What medications are
contraindicated?

A) Aminoglycosides
B) Fluoroquinolones
C) Magnesium sulfate
D) All of the above
Q231 Neurology
A woman with fluctuating ptosis and muscle weakness
has a positive “ice pack” test; labs show +acetylcholine
receptor antibody. What medications are
contraindicated?
Dx myasthenia gravis:
-Acetylcholine receptor antibody
-(edrophonium test – anticholinesterase inhibitor;
rapidly reverses symptoms); ice pack test [but pupils
A) Aminoglycosides unaffected because they are smooth muscle]
-**chest CT to evaluate for thymoma**
B) Fluoroquinolones
Tx myasthenia gravis
C) Magnesium sulfate-pyridostigmine (for symptomatic treatment)
D) All of the above
-prednisone, immunosuppressants
-IVIG, plasmapheresis for myasthenic crisis
Q232 Neurology
A man with proximal muscle weakness that improves
with activity is evaluated; repetitive nerve stimulation
reveals an incremental response. What is the
treatment?

A) Edaravone
B) 3,4-diaminopyridine + pyridostigmine
Q232 Neurology
A man with proximal muscle weakness that improves
with activity is evaluated; repetitive nerve stimulation
reveals an incremental response. What is the
treatment?

A) Edaravone
B) 3,4-diaminopyridine + pyridostigmine
Q232 Neurology
A man with proximal muscle weakness that improves
with activity is evaluated; repetitive nerve stimulation
reveals an incremental response. What is the
treatment?
ALS
“3,4- DAP”, got my muscles back
A) Edaravone
B) 3,4-diaminopyridine + pyridostigmine
Lambert-Eaton Myasthenic Syndrome
-proximal muscle weakness; improves with activity; depressed or absent deep tendon reflexes;
due to antibodies against presynaptic calcium channel; treat with 3,4 diaminopyridine (or
guanidine) + pyridostigmine; must evaluate for small cell lung cancer (60% of cases!)
Q233 Neurology
A woman with optic neuritis has the following MRI. How
are steroids given?

A) Orally
B) Intravenously
Q233 Neurology
A woman with optic neuritis has the following MRI. How
are steroids given?

A) Orally
B) Intravenously
Q233 Neurology
A woman with optic neuritis has the following MRI. How
are steroids given?
Multiple sclerosis
-diagnosis: MRI (multiple asymmetric, often
periventricular, white matter lesions [Dawson
fingers] especially in the corpus collosum. Active
A) Orally lesions enhance with gadolinium; CSF (lumbar
puncture) may reveal elevated IgG or at least 2
B) Intravenously oligoclonal bands not found in the serum
-Treatment: high dose IV steroids (acute
exacerbations); Interferon beta, Ocrelizumab,
sphingosine modifiers - Siponimod and
ozanimod for chronic maintenance/disease
modification; also treat symptoms (e.g., baclofen
or tizanidine for spasticity); amantadine for
fatigue; bethanechol for neurogenic bladder,
dalfampridine to improve walking distance
Q233 Neurology
A woman with optic neuritis has the following MRI. How
are steroids given?
Multiple sclerosis
-diagnosis: MRI (multiple asymmetric, often
periventricular, white matter lesions [Dawson
fingers] especially in the corpus collosum. Active
A) Orally lesions enhance with gadolinium; CSF (lumbar
puncture) may reveal elevated IgG or at least 2
B) Intravenously oligoclonal bands not found in the serum
-Treatment: high dose IV steroids (acute
Optic neuritis in MS occurs in 50% of patients;
exacerbations); Interferon beta, Ocrelizumab,
it is usually unilateral; has a good prognosis,
even without treatment; give steroids before
sphingosine modifiers - Siponimod and
confirmation with MRI!!! ozanimod for chronic maintenance/disease
modification; also treat symptoms (e.g., baclofen
or tizanidine for spasticity); amantadine for
fatigue; bethanechol for neurogenic bladder,
dalfampridine to improve walking distance
Q233 Neurology
A woman with optic neuritis has the following MRI. How
are steroids given?
Multiple sclerosis
-diagnosis: MRI (multiple asymmetric, often
periventricular, white matter lesions [Dawson
fingers] especially in the corpus collosum. Active
A) Orally lesions enhance with gadolinium; CSF (lumbar
puncture) may reveal elevated IgG or at least 2
B) Intravenously oligoclonal bands not found in the serum
-Treatment: high dose IV steroids (acute
Optic neuritis in MS occurs in 50% of patients;
exacerbations); Interferon beta, Ocrelizumab,
it is usually unilateral; has a good prognosis,
even without treatment; give steroids before
sphingosine modifiers - Siponimod and
confirmation with MRI!!! ozanimod for chronic maintenance/disease
modification; also treat symptoms (e.g., baclofen
Associated with vitamin D or tizanidine for spasticity); amantadine for
deficiency (all patients get
fatigue; bethanechol for neurogenic bladder,
vitamin D!), and EBV
dalfampridine to improve walking distance
Q233 Neurology
A woman with optic neuritis has the following MRI. How
are steroids given?
Multiple sclerosis
-diagnosis: MRI (multiple asymmetric, often
Internuclear ophthalmoplegia = unique to MS
periventricular, white matter lesions [Dawson
fingers] especially in the corpus collosum. Active
A) Orally lesions enhance with gadolinium; CSF (lumbar
puncture) may reveal elevated IgG or at least 2
B) Intravenously oligoclonal bands not found in the serum
-Treatment: high dose IV steroids (acute
Optic neuritis in MS occurs in 50% of patients;
exacerbations); Interferon beta, Ocrelizumab,
it is usually unilateral; has a good prognosis,
even without treatment; give steroids before
sphingosine modifiers - Siponimod and
confirmation with MRI!!! ozanimod for chronic maintenance/disease
modification; also treat symptoms (e.g., baclofen
Associated with vitamin D or tizanidine for spasticity); amantadine for
deficiency (all patients get
fatigue; bethanechol for neurogenic bladder,
vitamin D!), and EBV
dalfampridine to improve walking distance
Q234 Neurology
Guillain Barre Syndrome is characterized by
autoimmune peripheral demyelination, ascending
paralysis, and:

A) normal CSF protein


B) elevated CSF protein
Q234 Neurology
Guillain Barre Syndrome is characterized by
autoimmune peripheral demyelination, ascending
paralysis, and:

A) normal CSF protein


B) elevated CSF protein
Q234 Neurology
Guillain Barre Syndrome is characterized by
autoimmune peripheral demyelination, ascending
paralysis, and:

A) normal CSF protein


B) elevated CSF protein
GBS is a complete PNS disease!!! is associated with recent Campylobacter
infection, viral infection (*even acute HIV*); 85% make a complete or near-
complete recovery within 1 year; ascending paralysis and areflexia seen;
decreased nerve conduction seen due to diffuse demyelination; supported by
CSF > 55 (if normal CSF findings, may be paralysis (which is self-resolving after
tick is removed): Tx: IVIG, plasmapheresis, NOT steroids!!
Q234 “GBS? High CSF protein, YES!” Neurology
Guillain Barre Syndrome is characterized by
autoimmune peripheral demyelination, ascending
paralysis, and:

A) normal CSF protein


B) elevated CSF protein
GBS is a complete PNS disease!!! is associated with recent Campylobacter infection,
viral infection (*even acute HIV*); 85% make a complete or near-complete recovery within
1 year; ascending paralysis and areflexia seen; decreased nerve conduction seen due
to diffuse demyelination; supported by CSF > 55 (if normal CSF findings, may be
paralysis (which is self-resolving after tick is removed): Tx: IVIG, plasmapheresis, NOT
steroids!!
Q234 “GBS? High CSF protein, YES!” Neurology
Guillain Barre Syndrome is characterized by
autoimmune peripheral demyelination, ascending
paralysis, and:
Once GBS is suspected in a stable patient, must assess pulmonary function with SPIROMETRY (e.g., PIP, vital
capacity)! pulmonary function testing should be performed given risk for rapid progression of disease.

A) normal CSF protein


B) elevated CSF protein
GBS is a complete PNS disease!!! is associated with recent Campylobacter infection, viral
infection (*even acute HIV*); 85% make a complete or near-complete recovery within 1 year;
ascending paralysis and areflexia seen; decreased nerve conduction seen due to diffuse
demyelination; supported by CSF > 55 (if normal CSF findings, may be paralysis (which is self-
resolving after tick is removed): Tx: IVIG, plasmapheresis, NOT steroids!!
Q235 Neurology
An 11-year-old boy presents with gait ataxia, scoliosis,
and slurred speech; DNA testing shows GAA repeat. What
is the most common cause of death?

A) Renal dysfunction
B) Heart failure
Q235 Neurology
An 11-year-old boy presents with gait ataxia, scoliosis,
and slurred speech; DNA testing shows GAA repeat. What
is the most common cause of death?

A) Renal dysfunction
B) Heart failure
Q235 Neurology
An 11-year-old boy presents with gait ataxia, scoliosis,
and slurred speech; DNA testing shows GAA repeat. What
is the most common cause of death?

A) Renal dysfunction
B) Heart failure
Friedrich ataxia
-autosomal recessive, -GAA repeat causing abnormal
frataxin protein
-cardiac complications: Hypertrophic cardiomyopathy,
myocardial fibrosis
Q236 Neurology
A 52-year-old man presents with slowly progressive
weakness; he has increased reflexes in his left upper
extremity and later in his right; he has lower
fasciculations and atrophy but no sexual dysfunction.
What may delay disease progression?

A) Riluzole
B) Baclofen
Q236 Neurology
A 52-year-old man presents with slowly progressive
weakness; he has increased reflexes in his left upper
extremity and later in his right; he has lower
fasciculations and atrophy but no sexual dysfunction.
What may delay disease progression?

A) Riluzole
B) Baclofen
Q236 Neurology
A 52-year-old man presents with slowly progressive
weakness; he has increased reflexes in his left upper
extremity and later in his right; he has lower
fasciculations and atrophy but no sexual dysfunction.
What may delay disease progression?
ALS controls VOLUNTARY MUSCLE CONTROL

A) Riluzole (both UMN and LMN signs)! (does not affect the
five senses!)
B) Baclofen (Riluzole decreases glutamate levels, which are
thought to contribute to nerve damage;
Edaravone is also given [antioxidant properties])
Q237 Neurology
Why is MRI performed in ALS patients?

A) to diagnose ALS
B) to exclude cervical spondylosis
Q237 Neurology
Why is MRI performed in ALS patients?

A) to diagnose ALS
B) to exclude cervical spondylosis
Q237 Neurology
Why is MRI performed in ALS patients?
CT/MRI in ALS won’t show generally show
A) to diagnose ALS anything; diagnosis is primarily clinical
B) to exclude cervical spondylosis
Q237 Neurology
Why is MRI performed in ALS patients?
CT/MRI in ALS won’t show generally show anything;
A) to diagnose ALS diagnosis is primarily clinical

B) to exclude cervical spondylosis


ALS
-a chronic progressive degenerative disease of unknown etiology; loss of UMN and
LMN; almost always progresses to respiratory failure and death usually within 5 years
of diagnosis; asymmetric progressive weakness of arms, legs, diaphragm
-”bulbar onset” in 25% of patients (i.e., first signs are difficulty swallowing, loss of
tongue motility, or difficulty speaking); these signs suggest pathology above the
foramen magnum (also excludes cervical spondylosis)
-in case of CO2 retention, the answer is not intubation, but biPap!
Q238 Neurology
The time course of dementia in a patient with vascular
dementia is:

A) abrupt -
B) gradual -
C) stepwise -
Q238 Neurology
The time course of dementia in a patient with vascular
dementia is:

A) abrupt -
B) gradual -
C) stepwise -
Q238 Neurology
The time course of dementia in a patient with vascular
dementia is:

A) abrupt – Creutzfeldt-Jakob disease


B) gradual – Alzheimer's, Pick disease, Lewy Body
C) stepwise – VASCULAR DEMENTIA!
Vascular Dementia
-dementia associated with a history of stroke and cerebrovascular disease; stepwise decline in
cognitive functioning; risk factors: age, HTN, diabetes, embolic sources, and a history of stroke
-diagnosis: dementia + (2 of the following): focal neurologic signs (e.g., hemiparesis, pronator
drift), symptom onset that was stepwise/abrupt/related to stroke, MRI showing large lacunar infarct
burden in cortical/subcortical region
Q239 Neurology
48-year-old man presents with hypersexual behavior and
personality changes. What area of the brain is affected?

A) frontal lobe
B) temporal lobe
C) both A & B
Q239 Neurology
48-year-old man presents with hypersexual behavior and
personality changes. What area of the brain is affected?

A) frontal lobe
B) temporal lobe
C) both A & B
Q239 Neurology
48-year-old man presents with hypersexual behavior and
personality changes. What area of the brain is affected?

A) frontal lobe
B) temporal lobe
C) both A & B
-frontotemporal dementia, Pick disease,
average age of onset = 54
-other symptoms: speech disturbances, inattentiveness,
impulsivity, extrapyramidal symptoms -no treatment
Q240 Neurology
A treatable form of dementia caused by impaired CSF
resorption. Treatment?

A) symptomatic
B) CSF drainage
Q240 Neurology
A treatable form of dementia caused by impaired CSF
resorption. Treatment?

A) symptomatic
B) CSF drainage
Q240 Neurology
A treatable form of dementia caused by impaired CSF
resorption. Treatment?

A) symptomatic
B) CSF drainage
Q240 Neurology
A treatable form of dementia caused by impaired CSF
resorption. Treatment?

A) symptomatic
B) CSF drainage
NPH Classic triad:
-wacky (dementia)
wet (incontinence)
-wobbly (gait apraxia)
- [headaches and increased ICP
(papilledema) usually absent]
Q240 Neurology
A treatable form of dementia caused by impaired CSF
resorption. Treatment? MRI shows ventricular
enlargement out of proportion to
sulcal atrophy; LP reveals normal

A) symptomatic
CSF pressures (and can be
therapeutic); Tx: LP, shunt

B) CSF drainage
NPH Classic triad:
-wacky (dementia)
wet (incontinence)
-wobbly (gait apraxia)
- [headaches and increased ICP
(papilledema) usually absent]
Q241 Neurology
A patient presents with rapid progression dementia and a
myoclonus. CSF will show elevated:

A) Alpha synuclein
B) CSF 14-3-3
Q241 Neurology
A patient presents with rapid progression dementia and a
myoclonus. CSF will show elevated:

A) Alpha synuclein
B) CSF 14-3-3
Q241 Neurology
A patient presents with rapid progression dementia and a
myoclonus. CSF will show elevated:

A) Alpha synuclein
B) CSF 14-3-3
Creutzfeldt-Jakob Disease
-most common prion disease; -extremely rare form of dementia
-CSF shows elevated 14-3-3 protein and tau protein (but replaced
by real time quacking test!); definitive diagnosis only with brain
biopsy or autopsy; MRI/EEG may be helpful (periodic sharp wave
complexes); -no treatment; death within a year
Q242 Neurology
A 66-year-old man develop progressive loss of executive
function and visual-spatial processing. He then presents
with rigidity, tremor, and a shuffling gait. Autopsy will
reveal:

A) abnormal clumps of alpha-synuclein


B) prions
Q242 Neurology
A 66-year-old man develop progressive loss of executive
function and visual-spatial processing. He then presents
with rigidity, tremor, and a shuffling gait. Autopsy will
reveal:

A) abnormal clumps of alpha-synuclein


B) prions
Q242 Neurology
A 66-year-old man develop progressive loss of executive
function and visual-spatial processing. He then presents
with rigidity, tremor, and a shuffling gait. Autopsy will
reveal:
A) abnormal clumps of alpha-synuclein
Lewy Body Dementia Order is the KEY!!!!
-compare with Alzheimer’s disease in which memory loss is predominant and occurs
FIRST (e.g., and THEN hallucinations)
-compare with Parkinson’s; LBD is characterized by progressive cognitive changes,
visual hallucinations, and THEN Parkinson-like movement abnormalities
-Diagnosis: definitive diagnosis only with brain biopsy or autopsy (Lewy bodies –
alpha synuclein proteins within neurons)
Q243 Neurology
40-year-old man with gradual onset dementia,
purposeless dance-like movements and irritability.
His father also has these symptoms at age 45. MRI
will show:

A) Shrinkage of lateral ventricles


B) Caudate and putamen atrophy
Q243 Neurology
40-year-old man with gradual onset dementia,
purposeless dance-like movements and irritability.
His father also has these symptoms at age 45. MRI
will show:

A) Shrinkage of lateral ventricles


B) Caudate and putamen atrophy
Q243 Neurology
40-year-old man with gradual onset dementia,
purposeless dance-like movements and irritability.
His father also has these symptoms at age 45. MRI
will show:
Enlargement of lateral ventricles seen

A) Shrinkage of lateral ventricles


B) Caudate and putamen atrophy
Huntington Disease – “movement + emotion disorder”
-autosomal dominant disease involving CAG triplet repeat in the
HD gene; number of repeats typically expands in subsequent
generations; presents at 30-50 years of age; no cure
Q244 Neurology
The best initial treatment in Parkinson disease is:

A) Levodopa/carbidopa
B) Selegeline
C) COMT inhibitors
Q244 Neurology
The best initial treatment in Parkinson disease is:

A) Levodopa/carbidopa
B) Selegeline
C) COMT inhibitors
Q244 Neurology
The best initial treatment in Parkinson disease is:
-(Levodopa is a dopamine precursor that
can cross the BBB; Carbidopa blocks
A) Levodopa/carbidopaperipheral conversion of levodopa to prevent
B) Selegeline
levodopa side effects [nausea; vomiting];
may cause hallucinations, dizziness,
C) COMT inhibitors involuntary movements). Dopamine
agonists (e.g., ropinirole, bromocriptine)
may be used for treatment in early disease
too (can cause hypotension and confusion);
Anticholinergics (e.g., trihexyphenidyl or
benztropine) generally reserved for younger
patients whose primary symptom is tremor;
quetiapine given for psychosis
Q244 Neurology
The best initial treatment in Parkinson disease is:
-(Levodopa is a dopamine precursor that
can cross the BBB; Carbidopa blocks
A) Levodopa/carbidopa peripheral conversion of levodopa to

B) Selegeline may cause confusion


Next best treatment; prevent levodopa side effects [nausea;
vomiting]; may cause hallucinations,
and insomnia
C) COMT inhibitors dizziness, involuntary movements).
Dopamine agonists (e.g., ropinirole,
Entacapone/tolcapone; not given alone
but increase bioavailability of levodopa bromocriptine) may be used for treatment
in early disease too (can cause
hypotension and confusion); Anticholinergics
(e.g., trihexyphenidyl or benztropine) generally
reserved for younger patients whose primary
symptom is tremor (may cause dryness)
Q245 Neurology
A 44-year-old man has an urge to move his legs
especially at night; the urge is relieved by movement.
What is the treatment (besides addressing
underlying cause)?

A) Pramipexole
B) Beta blockers
Q245 Neurology
A 44-year-old man has an urge to move his legs
especially at night; the urge is relieved by movement.
What is the treatment (besides addressing
underlying cause)?

A) Pramipexole
B) Beta blockers
Q245 Neurology
A 44-year-old man has an urge to move his legs
especially at night; the urge is relieved by movement.
What is the treatment (besides addressing
underlying cause)? Restless leg
Syndrome
-associated with
neuropathy, iron
A) Pramipexole deficiency anemia
-treat with
B) Beta blockers gabapentin or
pregabalin
UptoDate: Dopamine agonists are seldom used -provide iron
for intermittent RLS because they have a delayed
onset of action (usually 90 to 120 minutes) and
are less helpful once symptoms have already
started.
Q246 Neurology
What is the most common adult primary brain
tumor?

A) Glioblastoma multiforme & meningioma


B) Astrocytomas & medulloblastomas
Q246 Neurology
What is the most common adult primary brain
tumor?

A) Glioblastoma multiforme & meningioma


B) Astrocytomas & medulloblastomas
Q246 Neurology
What is the most common adult primary brain
tumor?
in men in women
A) Glioblastoma multiforme & meningioma
B) Astrocytomas & medulloblastomas
in children
Q247 Neurology
A 60-year-old man presents with headaches and
dizziness. MRI shows a “butterfly glioma”. What is
the prognosis?

A) Good
B) Terrible
Q247 Neurology
A 60-year-old man presents with headaches and
dizziness. MRI shows a “butterfly glioma”. What is
the prognosis?

A) Good
B) Terrible
Q247 Neurology
A 60-year-old man presents with headaches and
dizziness. MRI shows a “butterfly glioma”. What is
the prognosis? Glioblastoma

A) Good
B) Terrible
<1 year from time of diagnosis
Q247 Neurology
A 60-year-old man presents with headaches and
dizziness. MRI shows a “butterfly glioma”. What is
the prognosis? Glioblastoma

Benign tumors include meningioma and vestibular schwannoma

A) Good
B) Terrible
<1 year from time of diagnosis
Q248 Neurology
Autosomal dominant disorder that affects many
organ systems, including the CNS and skin (e.g.,
ash-leaf lesions on the trunk, sebaceous adenomas
[see images]):

A) Tuberous sclerosis
B) Sturge Weber
Q248 Neurology
Autosomal dominant disorder that affects many
organ systems, including the CNS and skin (e.g.,
ash-leaf lesions on the trunk, sebaceous adenomas
[see images]):

A) Tuberous sclerosis
B) Sturge Weber
Q248 Neurology
Autosomal dominant disorder that affects many
organ systems, including the CNS and skin (e.g.,
ash-leaf lesions on the trunk, sebaceous adenomas
[see images]):
Neuro abnormalities (e.g., seizures), cutaneous abnormalities (e.g., sebaceous adenomas, ash-
leaf spots)

A) Tuberous sclerosis
B) Sturge Weber
Q248 Neurology
Autosomal dominant disorder that affects many
organ systems, including the CNS and skin (e.g.,
ash-leaf lesions on the trunk, sebaceous adenomas
[see images]):
Neuro abnormalities (e.g., seizures), cutaneous abnormalities (e.g., sebaceous adenomas, ash-
leaf spots)

A) Tuberous sclerosis
B) Sturge Weber
Neurocutaneous condition,
port-wine stain in the
trigeminal nerve distribution;
glaucoma, seizures,
intracranial calcifications
Q249 Neurology
Damage to the right optic
nerve would lead to:

A) Right anopia
B) Bitemporal
hemianopsia
C) Left homonymous
hemianopia
Q249 Neurology
Damage to the right optic
nerve would lead to:

A) Right anopia
B) Bitemporal
hemianopsia
C) Left homonymous
hemianopia
Q249 Neurology
Damage to the right optic
nerve would lead to:

A) Right anopia
B) Bitemporal If optic chiasm is
damaged
hemianopsia
C) Left homonymous
hemianopia is damaged
If right optic tract
Q250 Neurology
A man presents with sudden-onset headache and a
dilated pupil in his right eye that is nonreactive to
light. His right eye is hard to the touch. What
medication should be avoided?

A) NSAID
B) Atropine
Q250 Neurology
A man presents with sudden-onset headache and a
dilated pupil in his right eye that is nonreactive to
light. His right eye is hard to the touch. What
medication should be avoided?

A) NSAID
B) Atropine
Q250 Neurology
A man presents with sudden-onset headache and a
dilated pupil in his right eye that is nonreactive to
light. His right eye is hard to the touch. What
medication should be avoided?
Closed Angle Glaucoma
A) NSAID -disrupted flow of aqueous humor into the anterior chamber -> increased pressure in
the anterior chamber; predisposed patients have a more anterior lens against the iris

B) Atropine
-risk factors: family history, older age, Asian, prolonged pupillary dilation (e.g., being in dark
for long time), stress, uveitis; anticholinergics can cause it due to pupillary dilation
-presentation: extreme, sudden onset eye pain, blurred vision, headache, vomiting; hard,
red eye; dilated pupil, nonreactive to light
-dx: ocular tonometry (best initial), gonioscopy (gold standard)
-tx: emergency!! eyedrops (timilol, pilocarpine), mannitol; systemic acetazolamide – IV or oral,
do NOT give atropine or other meds that dilate pupils; laser peripheral iridotomy is curative
Q251 Neurology
A diabetic develops loss of visual acuity and
difficulty with seeing at night. What is the
treatment?

A) VEGF inhibitor
B) Surgical lens replacement
Q251 Neurology
A diabetic develops loss of visual acuity and
difficulty with seeing at night. What is the
treatment?

A) VEGF inhibitor
B) Surgical lens replacement
Q251 Neurology
A diabetic develops loss of visual acuity and
difficulty with seeing at night. What is the
treatment?

A) VEGF inhibitor
B) Surgical lens replacement
Cataracts are associated with diabetes,
hypertension, advanced age, and
exposure to radiation
Q252 Neurology
Painless, gradual loss of central vision with
fundoscopy revealing drusen (white/yellow
extracellular material)?

A) VEGF inhibitor
B) No current treatment
Q252 Neurology
Painless, gradual loss of central vision with
fundoscopy revealing drusen (white/yellow
extracellular material)?

A) VEGF inhibitor
B) No current treatment
Q252 Neurology
Painless, gradual loss of central vision with
fundoscopy revealing drusen (white/yellow
extracellular material)?
For exudative/vascular, “wet” macular degeneration; less
common, associated with rapid, severe vision damage

A) VEGF inhibitor
B) No current treatment
Atrophic “dry” macular degeneration – 80% of cases
(leading cause of permanent bilateral visual loss in the
US); “faces look blurry”; associated with smoking;
gradual vision loss; fundoscopy shows drusen; no
treatment currently available but vitamin C, vitamin E,
and zinc may slow disease progression
Q253 Neurology
Sudden onset flashing lights and blurred vision:

A) Central retinal artery occlusion

B) Central retinal vein occlusion

C) Retinal detachment
Q253 Neurology
Sudden onset flashing lights and blurred vision:

A) Central retinal artery occlusion

B) Central retinal vein occlusion

C) Retinal detachment
Q253 Neurology
Sudden onset flashing lights and blurred vision:

A) Central retinal artery occlusion


Sudden, painless, unilateral blindness; cherry red-spot on fovea; transient occlusion
= amaurosis fugax; tx: ocular massage, high flow o2; l thrombolysis within 8 hours

B) Central retinal vein occlusion


Rapid, painless vision loss; cotton wool spots (“Mars”),
associated with HTN; tx: Vegf inhibitors, laser photocoagulation

C) Retinal detachment
Sudden onset flashing lights; “curtain coming down over
eye:; exam shows gray, elevated retina; tell patient to lie
down and look at ceiling until ophthalmology arrives
Q254 Neurology
Café-au-lait spots + neurofibromas + axillary
freckling + Lisch nodules + scoliosis =

A) NF1
B) NF2
Q254 Neurology
Café-au-lait spots + neurofibromas + axillary
freckling + Lisch nodules + scoliosis =

A) NF1
B) NF2
Q254 Neurology
Café-au-lait spots + neurofibromas + axillary
freckling + Lisch nodules + scoliosis =

A) NF1 MUCH more common than


NF2! Clinically evident by
B) NF2 adolescence; no cure
-other findings: seizures and
optic gliomas (found in
children, can cause vision loss
as tumor expands)!!
Q255 Neurology
First-line for trigeminal neuralgia is:

A) Valproic acid
B) Carbamazepine
Q255 Neurology
First-line for trigeminal neuralgia is:

A) Valproic acid
B) Carbamazepine
Q255 Neurology
First-line for trigeminal neuralgia is:

A) Valproic acid – can be used if carbamazepine doesn’t work


B) Carbamazepine
Note: bilateral trigeminal
neuralgia is sometimes seen
in multiple sclerosis

Pathophysiology of TN: compression of the


trigeminal nerve root as it enters the pons, leading
to demyelination and atrophy of the nerve (but also
from a neoplastic growth or a MS plaque)
Q256 Neurology
A man with schizophrenia is treated with
risperidone. He now has gynecomastia and
decreased libido. This occurred due to decreased
dopamine in the:

A) mesolimbic pathway
B) nigrostriatal pathway
C) tuberoinfundibular pathway
Q256 Neurology
A man with schizophrenia is treated with
risperidone. He now has gynecomastia and
decreased libido. This occurred due to decreased
dopamine in the:

A) mesolimbic pathway
B) nigrostriatal pathway
C) tuberoinfundibular pathway
Q256 Neurology
A man with schizophrenia is treated with
risperidone. He now has gynecomastia and
decreased libido. This occurred due to decreased
dopamine in the:

A) mesolimbic pathway – antipsychotic effect


B) nigrostriatal pathway – extra-pyramidal symptoms
C) tuberoinfundibular pathway – dopamine antagonism here leads to
increased prolactin → gynecomastia and decreased libido (amenorrhea in females)
Q257 Neurology
A 71-year-old woman with a history of CAD and HTN
experiences painless, sudden, transient monocular
vision loss; she has had no focal weakness,
numbness, or pain. What happened?

A) embolization
B) vasculitis induced vision loss
Q257 Neurology
A 71-year-old woman with a history of CAD and HTN
experiences painless, sudden, transient monocular
vision loss; she has had no focal weakness,
numbness, or pain. What happened?

A) embolization
B) vasculitis induced vision loss
Q257 Neurology
A 71-year-old woman with a history of CAD and HTN
experiences painless, sudden, transient monocular
vision loss; she has had no focal weakness,
numbness, or pain. What happened?
she most likely experienced amaurosis fugax - painless, sudden, and transient monocular vision
loss; most commonly results from carotid artery plaque embolizing in the retinal artery
A) embolization
B) vasculitis induced vision loss
Q257 Neurology
A 71-year-old woman with a history of CAD and HTN
experiences painless, sudden, transient monocular
vision loss; she has had no focal weakness,
numbness, or pain. What happened?
she most likely experienced amaurosis fugax - painless, sudden, and transient monocular vision
loss; most commonly results from carotid artery plaque embolizing in the retinal artery
A) embolization
B) vasculitis induced vision loss
In Giant cell arteritis, there are associated
symptoms (e.g., headache, jaw claudication, fever)
Q258 Neurology
A 76-year-old man has progressive hearing loss,
especially in noisy places; he also hears "buzzing" in
both ears. Otoscopic exam is normal. A tuning fork
placed in the middle of the head is heard equally
bilaterally and AC > BC. What is the diagnosis?

A) Otosclerosis
B) Presbycusis
Q258 Neurology
A 76-year-old man has progressive hearing loss,
especially in noisy places; he also hears "buzzing" in
both ears. Otoscopic exam is normal. A tuning fork
placed in the middle of the head is heard equally
bilaterally and AC > BC. What is the diagnosis?

A) Otosclerosis
B) Presbycusis
Q258 Neurology
A 76-year-old man has progressive hearing loss,
especially in noisy places; he also hears "buzzing" in
both ears. Otoscopic exam is normal. A tuning fork
placed in the middle of the head is heard equally
bilaterally and AC > BC. What is the diagnosis?
Otosclerosis = stiffening of the ossicles;
Weber lateralizes to the affected ear,
A) Otosclerosis where BC > AC

B) Presbycusis sensorineural hearing loss; most likely presbycusis (age related


Weber heard well bilaterally, AC > BC suggests bilateral symmetric

hearing loss – no nystagmus/vertigo); affects 50% of adults by age


75. No cure! Hearing aids can be helpful; family education.
Q259 Neurology
MRI of a 20-year-old male shows inferior
displacement of the cerebellum, as shown. What is
he at risk for?

A) Vestibular schwannoma
B) Cervical syringomyelia
Q259 Neurology
MRI of a 20-year-old male shows inferior
displacement of the cerebellum, as shown. What is
he at risk for?

A) Vestibular schwannoma
B) Cervical syringomyelia
Q259 Neurology
MRI of a 20-year-old male shows inferior
displacement of the cerebellum, as shown. What is
he at risk for?

A) Vestibular schwannoma
B) Cervical syringomyelia
Cervical syringomyelia occurs in 30% of patients
with Chiari malformation
(condition is sometimes asymptomatic; sometimes
patients present with occipital headache)
Q260 Neurology
A patient with bilateral retinal hemangioblastomas
has a family history of an adrenal tumor. What is
the diagnosis?

A) NF1
B) Von Hippel Lindau
Q260 Neurology
A patient with bilateral retinal hemangioblastomas
has a family history of an adrenal tumor. What is
the diagnosis?

A) NF1
B) Von Hippel Lindau
Q260 Neurology
A patient with bilateral retinal hemangioblastomas
has a family history of an adrenal tumor. What is
the diagnosis?

A) NF1
B) Von Hippel Lindau
Von Hippel-Lindau disease is an autosomal dominant
condition caused by a mutation of a tumor suppressor
gene; leads to retinal hemangioblastomas,
pheochromocytomas, and renal cell carcinomas.
Q261 Neurology
A 23-year-old woman presents with hearing loss
and balancing problems. Her mother had similar
symptoms. What is the most likely diagnosis?

A) MS
B) NF2
Q261 Neurology
A 23-year-old woman presents with hearing loss
and balancing problems. Her mother had similar
symptoms. What is the most likely diagnosis?

A) MS
B) NF2
Q261 Neurology
A 23-year-old woman presents with hearing loss
and balancing problems. Her mother had similar
symptoms. What is the most likely diagnosis?

A) MS
B) NF2
NF2 = bilateral vestibular schwannomas,
imbalance (especially with eyes closed);
usually presents in 20's
Q262 Neurology
A 3-year-old boy has refractory epilepsy,
developmental delay, hypopigmented macules,
and MRI is shown. Which is associated with his
condition?

A) Renal angiomyolipoma
B) Optic nerve glioma
Q262 Neurology
A 3-year-old boy has refractory epilepsy,
developmental delay, hypopigmented macules,
and MRI is shown. Which is associated with his
condition?

A) Renal angiomyolipoma
B) Optic nerve glioma
Q262 Neurology
A 3-year-old boy has refractory epilepsy,
developmental delay, hypopigmented macules,
and MRI is shown. Which is associated with hisnodule
Subependymal

condition?
(the classic CNS tumor in TS)

A) Renal angiomyolipoma
B) Optic nerve glioma
Q263 Neurology
A 9-year-old girl has spots on her trunk and her
eye is shown. She is at risk for:

A) Peripheral nerve sheath tumors


B) Renal angiomyolipoma
C) Retinal hemangioblastoma
Q263 Neurology
A 9-year-old girl has spots on her trunk and her
eye is shown. She is at risk for:

A) Peripheral nerve sheath tumors


B) Renal angiomyolipoma
C) Retinal hemangioblastoma
Q263 Neurology
A 9-year-old girl has spots on her trunk and her
eye is shown. She is at risk for:

A) Peripheral nerve sheath tumors


B) Renal angiomyolipoma - TS Lisch nodules

C) Retinal hemangioblastoma - VHL


Q264 Neurology
A 5-year-old girl is brought to the ED for a
cerebrovascular accident. She has fair skin, long,
thin arms and legs with joint hyperlaxity; she
also has a history of eye problems and
developmental delays. What is the diagnosis?

A) Marfan syndrome
B) Homocystinuria
Q264 Neurology
A 5-year-old girl is brought to the ED for a
cerebrovascular accident. She has fair skin, long,
thin arms and legs with joint hyperlaxity; she
also has a history of eye problems and
developmental delays. What is the diagnosis?

A) Marfan syndrome
B) Homocystinuria
Q264 Neurology
A 5-year-old girl is brought to the ED for a
cerebrovascular accident. She has fair skin, long,
thin arms and legs with joint hyperlaxity; she
also has a history of eye problems and
developmental delays. What is the diagnosis?
Both Marfan and Homocystinuria ("marfanoid
habitus") can involve tall stature, arachnodactyly,
A) Marfan syndrome joint hyperlaxity, and pectus deformity; but
whereas Marfan involves aortic root dilation and
B) Homocystinuria upward lens dislocation, homocystinuria involves
intellectual disability, fair complexion, downward
dislocation, and thrombosis (CVA).
Q265 Neurology
A 20-year-old boy with behavioral changes,
neurologic abnormalities, and hepatomegaly
should be assessed for:

A) serum ceruloplasmin
B) nerve conduction studies
Q265 Neurology
A 20-year-old boy with behavioral changes,
neurologic abnormalities, and hepatomegaly
should be assessed for:

A) serum ceruloplasmin
B) nerve conduction studies
Q265 Neurology
A 20-year-old boy with behavioral changes,
neurologic abnormalities, and hepatomegaly
should be assessed for:

A) serum ceruloplasmin
B) nerve conduction studies
Wilson disease
-rare, autosomal recessive disease; abnormal copper deposition in tissues (liver, basal
ganglia, corneas); young children develop liver disease (asymptomatic to hepatomegaly
to liver failure); also neuropsychiatric disease (with dysarthria, hyperreflexia); academic
decline, irritability, depression; best initial test - slit lamp test for kaiser Fleisher rings;
most accurate: biopsy or 24 hour copper excretion with penicillamine
Q266 Neurology
a 17-year-old male has muscle pain and wasting
along with small testicles. How is his condition
inherited?

A) Mitochondrial
B) Autosomal dominant
C) X-linked dominant
Q266 Neurology
a 17-year-old male has muscle pain and wasting
along with small testicles. How is his condition
inherited?

A) Mitochondrial
B) Autosomal dominant
C) X-linked dominant
Q266 Neurology
a 17-year-old male has muscle pain and wasting
along with small testicles. How is his condition
inherited?

A) Mitochondrial
B) Autosomal dominant
C) X-linked dominant
Myotonic dystrophy - autosomal dominant trinucleotide repeat (CTG);
presents in adolescence/early adulthood - progressive weakness (facial
and hand muscles), dysphagia, conduction anomalies, testicular atrophy
Q267 Neurology
A 6-month-old girl who was born preterm has
decreased use of her left arm; MRI of the brain
shows white matter injury near the right lateral
ventricle. What most likely led to this condition?

A) Transplacental infection
B) Preterm delivery
Q267 Neurology
A 6-month-old girl who was born preterm has
decreased use of her left arm; MRI of the brain
shows white matter injury near the right lateral
ventricle. What most likely led to this condition?

A) Transplacental infection
B) Preterm delivery
Q267 Neurology
A 6-month-old girl who was born preterm has
decreased use of her left arm; MRI of the brain
shows white matter injury near the right lateral
ventricle. What most likely led to this condition?

A) Transplacental infection
B) Preterm delivery
Signs of cerebral palsy (nonprogressive motor dysfunction) - motor delay (e.g., not rolling over at
age 4 months), early hand preference (at less than 1 year), persistent neonatal reflexes, abnormal
tone; greatest in premature infants due to ischemia and necrosis in the poorly perfused areas
adjacent to the lateral ventricles
Q268 Neurology
A preterm-born 10-month-old girl pulls herself to
standing using her arms but must drag her legs. She
says "mommy" and "daddy". Bilateral lower
extremities are hypertonic with notable resistance
to passive extension. What is the diagnosis?

A) Myotonic dystrophy
B) Cerebral palsy
C) Rett syndrome
Q268 Neurology
A preterm-born 10-month-old girl pulls herself to
standing using her arms but must drag her legs. She
says "mommy" and "daddy". Bilateral lower
extremities are hypertonic with notable resistance
to passive extension. What is the diagnosis?

A) Myotonic dystrophy
B) Cerebral palsy
C) Rett syndrome
Q268 Neurology
A preterm-born 10-month-old girl pulls herself to
standing using her arms but must drag her legs. She
says "mommy" and "daddy". Bilateral lower
extremities are hypertonic with notable resistance
to passive extension. What is the diagnosis?
Can be seen in neonates but exam would show muscle wasting; hyperreflexia would not be
present; sustained grip would be seen
A) Myotonic dystrophy
The first sign of cerebral palsy is often gross motor delay;
spastic cerebral palsy is a subtype which involves resistance

B) Cerebral palsy
to passive muscle stretch. Hypertonia and hyperreflexia
predominantly involve the lower extremities. May see child

C) Rett syndrome Abnormal gait, repetitive hand movements; microcephaly, no hyperreflexia


pull themselves up by their hands.
Q269 Neurology
A patient with intracerebral hemorrhage becomes
unresponsive; CT of the head shows right-sided
intracerebral hemorrhage with a 7.0-mm midline
shift. What is the next step?

A) mannitol
B) intubation and ventilation
Q269 Neurology
A patient with intracerebral hemorrhage becomes
unresponsive; CT of the head shows right-sided
intracerebral hemorrhage with a 7.0-mm midline
shift. What is the next step?

A) mannitol
B) intubation and ventilation
Q269 Neurology
A patient with intracerebral hemorrhage becomes
unresponsive; CT of the head shows right-sided
intracerebral hemorrhage with a 7.0-mm midline
shift. What is the next step?

A) mannitol
B) intubation and ventilation
In brain herniation, respiratory failure can develop quickly; intubate
and ventilate immediately! (this comes before reducing intracranial
pressure with mannitol, or other measures)
Q270 Neurology
A 16-year-old girl develops progressive night
blindness and peripheral vision loss. Funduscopic
exam shows retinal vessel attenuation. What is the
diagnosis?

A) Vitamin A deficiency
B) Retinitis pigmentosa
Q270 Neurology
A 16-year-old girl develops progressive night
blindness and peripheral vision loss. Funduscopic
exam shows retinal vessel attenuation. What is the
diagnosis?

A) Vitamin A deficiency
B) Retinitis pigmentosa
Q270 Neurology
A 16-year-old girl develops progressive night
blindness and peripheral vision loss. Funduscopic
exam shows retinal vessel attenuation. What is the
diagnosis?

A) Vitamin A deficiency
B) Retinitis pigmentosa
Q270 Neurology
A 16-year-old girl develops progressive night
blindness and peripheral vision loss. Funduscopic
exam shows retinal vessel attenuation. What is the
diagnosis?
Vitamin A deficiency - xerophthalmia (excessive dryness of the
cornea and conjunctivae), night blindness

A) Vitamin A deficiency
B) Retinitis pigmentosa
Q271 Neurology
A 60-year-old man with nausea,
vomiting, and AMS presents with a
brain hemorrhage as shown. What is
the cause?

A) Ruptured berry aneurysm


B) Ruptured AV malformation
C) Hypertensive intracerebral
hemorrhage
Q271 Neurology
A 60-year-old man with nausea,
vomiting, and AMS presents with a
brain hemorrhage as shown. What is
the cause?

A) Ruptured berry aneurysm


B) Ruptured AV malformation
C) Hypertensive intracerebral
hemorrhage
Q271 Neurology
A 60-year-old man with nausea,
vomiting, and AMS presents with a
brain hemorrhage as shown. What is
the cause?

A) Ruptured berry aneurysm


B) Ruptured AV malformation
C) Hypertensive intracerebral This is a basal ganglia intraparenchymal
hemorrhage - due mostly to hypertensive
hemorrhage vasculopathy.
Q271 Neurology
A 60-year-old man with nausea,
vomiting, and AMS presents with a
brain hemorrhage as shown. What is
the cause?
(Ruptured berry aneurysms are seen in subarachnoid hemorrhage; AV
malformation rupture seen in mostly children and are quite uncommon.)
A) Ruptured berry aneurysm
B) Ruptured AV malformation
C) Hypertensive intracerebral This is a basal ganglia intraparenchymal
hemorrhage - due mostly to hypertensive
hemorrhage vasculopathy.
Q272 Neurology
Severe onset headache with an associated
oculomotor nerve palsy (e.g., right sided ptosis,
right pupil bigger) is most associated with:

A) Cavernous sinus thrombosis


B) Subarachnoid hemorrhage
Q272 Neurology
Severe onset headache with an associated
oculomotor nerve palsy (e.g., right sided ptosis,
right pupil bigger) is most associated with:

A) Cavernous sinus thrombosis


B) Subarachnoid hemorrhage
Q272 Neurology
Severe onset headache with an associated
oculomotor nerve palsy (e.g., right sided ptosis,
right pupil bigger) is most associated with:
Cavernous sinus thrombosis usually
presents with fever, headache, and
A) Cavernous sinus thrombosis periorbital swelling; also, there is usually
palsy of CN III, CN IV, and CN VI (all 3 nerves)

B) Subarachnoid hemorrhage resulting in an extensive ophthalmoplegia.

Oculomotor nerve palsy (ONP) with


subarachnoid hemorrhage occurs usually when
oculomotor nerve is compressed by growing or
budding of posterior communicating artery
(PcoA) aneurysm.
Q273 Neurology
A woman on warfarin develops a brain
hemorrhage. INR = 6. Platelets = 160,000. What
is the next step?
A) Tranexamic acid
B) Protamine sulfate
C) Platelets
D) Prothrombin complex concentrate
Q273 Neurology
A woman on warfarin develops a brain
hemorrhage. INR = 6. Platelets = 160,000. What
is the next step?
A) Tranexamic acid
B) Protamine sulfate
C) Platelets
D) Prothrombin complex concentrate
Q273 Neurology
A woman on warfarin develops a brain
hemorrhage. INR = 6. Platelets = 160,000. What is
the next step?
A) Tranexamic acid - for heavy menstrual bleeding or to prevent excess blood loss during
certain types of surgery; **and for TBI to prevent brain hemorrhage**
B) Protamine sulfate - used for heparin reversal, not warfarin associated hemorrhage
C) Platelets - consider in patients with low platelets or on antiplatelet therapy at time of
brain bleed
D) Prothrombin complex concentrate - Intracerebral hemorrhage is a feared complication
of warfarin. immediately reverse with IV vitamin K and PCC
Q274 Neurology
An 85-year-old woman with a recent of falls
presents with a 2-week history of somnolence
and confusion. She has also developed a mild
headache. What is the diagnosis?

A) Internal capsule infarction


B) Subdural hematoma
Q274 Neurology
An 85-year-old woman with a recent of falls
presents with a 2-week history of somnolence
and confusion. She has also developed a mild
headache. What is the diagnosis?

A) Internal capsule infarction


B) Subdural hematoma
Q274 Neurology
An 85-year-old woman with a recent of falls
presents with a 2-week history of somnolence
and confusion. She has also developed a mild
headache. What is the diagnosis?

A) Internal capsule infarction


B) Subdural hematoma
There are both acute and chronic subdural hematomas. Chronic
ones present insidiously, weeks after initial injury. Symptoms
includes headache, somnolence, and confusion.
Q275 Neurology
A 43-year-old woman presents with sudden
onset severe headache with photopia and
vomiting. Noncontract CT is normal. What is the
next step?

A) MRI
B) Lumbar puncture
Q275 Neurology
A 43-year-old woman presents with sudden
onset severe headache with photopia and
vomiting. Noncontract CT is normal. What is the
next step?

A) MRI
B) Lumbar puncture
Q275 Neurology
A 43-year-old woman presents with sudden
onset severe headache with photopia and
vomiting. Noncontract CT is normal. What is the
next step?

A) MRI
B) Lumbar puncture
Patients with thunderclap headache should get urgent CT; however, it may
show nothing as blood degrades (esp. after 6 hours); so next step in lumbar
puncture (elevated opening pressure, xanthochromia, and high RBC count).
[MRI has similar sensitivity to CT scan so it wouldn't be useful here]
Q276 Neurology
A 70-year-old man presents with sudden complete
vision loss of his left eye after 24 hours of blurry
vision. He has no weakness or sensory changes,
nausea or vomiting, but malaise and fatigue. The
lateral scalp is tender. What should be given?

A) intravenous acetazolamide
B) IV steroids, then oral steroids for several
months
Q276 Neurology
A 70-year-old man presents with sudden complete
vision loss of his left eye after 24 hours of blurry
vision. He has no weakness or sensory changes,
nausea or vomiting, but malaise and fatigue. The
lateral scalp is tender. What should be given?

A) intravenous acetazolamide
B) IV steroids, then oral steroids for several
months
Q276 Neurology
A 70-year-old man presents with sudden complete
vision loss of his left eye after 24 hours of blurry
vision. He has no weakness or sensory changes,
nausea or vomiting, but malaise and fatigue. The
lateral scalp is tender. What should be given?
Acetazolamide is used for closed angle glaucoma (acute vision Remember to look out for long term steroid adverse
loss and headache); ocular pain, nausea, and vomiting effects!! E.g., Glucocorticoid-induced myopathy -
painless proximal muscle weakness, which is more
A) intravenous acetazolamide prominent in the lower extremities, with no muscle
inflammation or tenderness, and normal CK and ESR

B) IV steroids, then oral steroids for several


months before permanent loss; oral steroids for months to prevent worsening and prevent damage to other eye;
Concern for Giant Cell Arteritis (women>men) – most patients have temporary vision loss (amaurosis fugax)
Q277 Neurology
A hypertensive woman develops right sided
hemiparesis and then hours later develops
vomiting and headache. Her pulse is nor 52. What
is the etiology?

A) Cardiogenic embolus
B) Carotid artery atherosclerosis
C) Intracerebral hemorrhage
Q277 Neurology
A hypertensive woman develops right sided
hemiparesis and then hours later develops
vomiting and headache. Her pulse is now 52. What
is the etiology?

A) Cardiogenic embolus
B) Carotid artery atherosclerosis
C) Intracerebral hemorrhage
Q277 Neurology
A hypertensive woman develops right sided
hemiparesis and then hours later develops
vomiting and headache. Her pulse is nor 52. What
is the etiology?
Hemorrhagic stroke tends to have focal symptoms (e.g., hemiparesis) that
rapidly progresses to signs of elevated ICP; get urgent non-contrast CT
scan! (embolic stroke - onset is abrupt and is usually maximal at onset)

C) Intracerebral hemorrhage
Q278 Neurology
A 73-year-old woman with hypertension has
intermittent transient right eye vision loss. She
describes a "curtain coming down over my eye".
What will reveal the diagnosis?

A) Ocular tonometry
B) Neck ultrasound
Q278 Neurology
A 73-year-old woman with hypertension has
intermittent transient right eye vision loss. She
describes a "curtain coming down over my eye".
What will reveal the diagnosis?

A) Ocular tonometry
B) Neck ultrasound
Q278 Neurology
A 73-year-old woman with hypertension has
intermittent transient right eye vision loss. She
describes a "curtain coming down over my eye".
What will reveal the diagnosis?
(Tonometry is for acute closure glaucoma - painful monocular
vision loss and headache/nausea/vomiting)

A) Ocular tonometry
B) Neck ultrasound Most common cause of amaurosis fugax (painless,
transient, monocular vision loss) is retinal ischemia
due to an atherosclerotic embolus originating from
the ipsilateral carotid artery. So next step is
ultrasound of neck.
Q279 Neurology
A patient with Alzheimer’s has progressive
confusion and lethargy over the last few hours; CT
shows a hematoma in the right parietal lobe. This
is due to:

A) cerebral amyloid angiopathy


B) subdural hematoma
Q279 Neurology
A patient with Alzheimer’s has progressive
confusion and lethargy over the last few hours; CT
shows a hematoma in the right parietal lobe. This
is due to:

A) cerebral amyloid angiopathy


B) subdural hematoma
Q279 Neurology
Cerebral amyloid angiopathy is the most common cause of
lobar intracranial hemorrhage and 2nd most common cause
(after HTN) of intracranial hemorrhage;
-CAA associated with Alzheimer dementia;
-tx: the same as other cause of ICH - reverse anticoagulation,
blood pressure control, normalize intracranial pressure

A) cerebral amyloid angiopathy


B) subdural hematoma
Banana shaped on CT scan
Q280 Neurology
A lady tilts her head back to the
right and develops loss of right face
pain and temperature as well as
nystagmus and ataxia. What
happened?

A) right vertebral artery dissection


B) otolith dislodgement
Q280 Neurology
A lady tilts her head back to the
right and develops loss of right face
pain and temperature as well as
nystagmus and ataxia. What
happened?

A) right vertebral artery dissection


B) otolith dislodgement
Q280 Neurology
A lady tilts her head back to the
right and develops loss of right face
pain and temperature as well as
nystagmus and ataxia. What
happened? Vertigo + hemisensory loss can be caused by
posterior circulation stroke precipitated by a
vertebral artery dissection
A) right vertebral artery dissection
B) otolith dislodgement
BPPV = vertigo and nystagmus (triggered by head movement), but episodes are brief
and there are no other neurologic findings
Q281 Neurology
A man who experienced a stroke shaves only the
right side of his face; when asked to stick out his
left arm, he sticks out his right. Where is the
stroke?

A) Right frontal cortex


B) Right parietal cortex
Q281 Neurology
A man who experienced a stroke shaves only the
right side of his face; when asked to stick out his
left arm, he sticks out his right. Where is the
stroke?

A) Right frontal cortex


B) Right parietal cortex
Q281 Neurology
A man who experienced a stroke shaves only the
right side of his face; when asked to stick out his
left arm, he sticks out his right. Where is the
stroke?

A) Right frontal cortex


B) Right parietal cortex
Damage to the nondominant (right) parietal lobe can
lead to hemineglect (unawareness of left side of world
and/or body) - sensory/motor/concepts.
Q282 Neurology
Vertigo, ataxia, loss of pain and temperature on
left side of face and right side of body =

A) Lesion in left medulla


B) Lesion in left midbrain
Q282 Neurology
Vertigo, ataxia, loss of pain and temperature on
left side of face and right side of body =

A) Lesion in left medulla


B) Lesion in left midbrain
Q282 Neurology
Vertigo, ataxia, loss of pain and temperature on
left side of face and right side of body =
This is consistent with Wallenberg syndrome (a lateral MEDULLARLY infarct)
A) Lesion in left medulla
B) Lesion in left midbrain
Q282 Neurology
Vertigo, ataxia, loss of pain and temperature on
left side of face and right side of body =
This is consistent with Wallenberg syndrome (a lateral MEDULLARLY infarct)
A) Lesion in left medulla
B) Lesion in left midbrain
Q283 Neurology
What does this CT scan
show?

A) Cerebral venous
thrombosis
B) Subarachnoid
hemorrhage
Q283 Neurology
What does this CT scan
show?

A) Cerebral venous
thrombosis
B) Subarachnoid
hemorrhage
Q283 Neurology
What does this CT scan
show?

A) Cerebral venous
thrombosis unremarkable
diagnosis with MRI; CT

B) Subarachnoid
hemorrhage
CT scan shows bleeding around brainstem and basal
cisterns; (remember - CT scan before LP!) Note:
nontraumatic subarachnoid hemorrhage is most
commonly due to ruptured saccular aneurysm
Q284 Neurology
A man with sudden vision loss in one
eye has the following fundoscopic
image. What is the diagnosis?

A) Retinal vein occlusion


B) Retinal artery occlusion
Q284 Neurology
A man with sudden vision loss in one
eye has the following fundoscopic
image. What is the diagnosis?

A) Retinal vein occlusion


B) Retinal artery occlusion
Q284 Neurology
A man with sudden vision loss in one Central retinal vein occlusion -

eye has the following fundoscopic


dilated/tortuous veins; cotton
wool spots

image. What is the diagnosis?

A) Retinal vein occlusion


B) Retinal artery occlusion
Central retinal artery occlusion - cherry red spot (central fovea) –
Remember to get neck ultrasound!
Q285 Neurology
A 64-year-old lady with a history of HTN
presents with sudden onset headache with
nausea, vomiting, "room is spinning"
feeling, and non-fatigable nystagmus.
What is the next step?

A) Epley maneuver
B) CT scan
Q285 Neurology
A 64-year-old lady with a history of HTN
presents with sudden onset headache with
nausea, vomiting, "room is spinning"
feeling, and non-fatigable nystagmus.
What is the next step?

A) Epley maneuver
B) CT scan
Q285 Neurology
A 64-year-old lady with a history of HTN
presents with sudden onset headache with
nausea, vomiting, "room is spinning"
feeling, and non-fatigable nystagmus.
What is the next step?

A) Epley maneuver
B) CT scan nystagmus (i.e., more than one minute) is more associated with
Vascular risk factors + acute onset headache with non-fatigable

stroke/hemorrhage of the cerebellum (vs. BPPV where nystagmus is


fatigable and no acute onset headache)
Q286 Neurology
A 62-year-old woman falls due to imbalance; she
has a history of HTN; she has vertigo with
movement of the head and dysmetria (she has an
abnormal heal-to-shin testing on the left). What is
the next step?

A) Tilt table test


B) Carotid ultrasound
C) IV alteplase
Q286 Neurology
A 62-year-old woman falls due to imbalance; she
has a history of HTN; she has vertigo with
movement of the head and dysmetria (she has an
abnormal heal-to-shin testing on the left). What is
the next step?

A) Tilt table test


B) Carotid ultrasound
C) IV alteplase
Q286 Neurology
A 62-year-old woman falls due to imbalance; she
has a history of HTN; she has vertigo with
movement of the head and dysmetria (she has an
abnormal heal-to-shin testing on the left). What is
the next step?

A) Tilt table test


B) Carotid ultrasound
Not for emergency situations! But to assess
e.g., for carotid endarterectomy

C) IV alteplase
Q287 Neurology
A patient presents with stroke (ischemic as per CT
scan) which occurred 7 hours ago. He has disabling
neurologic symptoms. What is the next step?

A) tPA
B) heparin
C) nicardipine
D) CT angiography of the head and neck
Q287 Neurology
A patient presents with stroke (ischemic as per CT
scan) which occurred 7 hours ago. He has disabling
neurologic symptoms. What is the next step?

A) tPA
B) heparin
C) nicardipine
D) CT angiography of the head and neck
Q287 Neurology
A patient presents with stroke (ischemic as per CT
scan) which occurred 7 hours ago. He has disabling
neurologic symptoms. What is the next step?

A) tPA
B) heparin
C) nicardipine
D) CT angiography of the head and neck
It’s been more than 24 hours so he’s no longer a candidate for tPa, but it’s within 24 hours so he’s still eligible for mechanical
thrombectomy – if there is a large thrombus vessel occlusion found. (if outside of 24 hours, just give antiplatelet therapy)
Q288 Neurology
A patient with ischemic stroke 2 hours ago. CT
angiography shows a flow void in the left MCA.
What is the next step?

A) tPA
B) tPA and thrombectomy
Q288 Neurology
A patient with ischemic stroke 2 hours ago. CT
angiography shows a flow void in the left MCA.
What is the next step?

A) tPA
B) tPA and thrombectomy
Q288 Neurology
A patient with ischemic stroke 2 hours ago. CT
angiography shows a flow void in the left MCA.
What is the next step?

A) tPA
B) tPA and thrombectomy
In patients with ischemic stroke, eligibility for thrombolysis
and mechanical thrombectomy is considered independently;
i.e., if within 4.5 hours for thrombolysis and within 24 hours
for thrombectomy, and CT angiography showing MCA
occlusion - both are indicated!
Q289 Neurology
Spontaneous cerebellar hemorrhage presents with
headache, nausea/vomiting, nystagmus, and:

A) Hemiparesis
B) Unilateral hemiataxia
Q289 Neurology
Spontaneous cerebellar hemorrhage presents with
headache, nausea/vomiting, nystagmus, and:

A) Hemiparesis
B) Unilateral hemiataxia
Q289 Neurology
Spontaneous cerebellar hemorrhage presents with
headache, nausea/vomiting, nystagmus, and:

A) Hemiparesis
B) Unilateral hemiataxia
-seen in patients with hypertension
-ataxia seen on ipsilateral side of the
hemorrhage
Q290 Neurology
Child with fever, irritability, vomiting, and
photophobia; CSF shows mildly elevated WBC's,
elevated protein, and normal glucose. What is the
cause?

A) Cryptococcus
B) Hemophilus
C) Listeria
D) Mumps
Q290 Neurology
Child with fever, irritability, vomiting, and
photophobia; CSF shows mildly elevated WBC's,
elevated protein, and normal glucose. What is the
cause?

A) Cryptococcus
B) Hemophilus
C) Listeria
D) Mumps
Q290 Neurology
Child with fever, irritability, vomiting, and
photophobia; CSF shows mildly elevated WBC's,
elevated protein, and normal glucose. What is the
cause?

A) Cryptococcus
B) Hemophilus
C) Listeria
D) Mumps elevated protein, but normal glucose levels
Of the choices, only viruses (mumps) have elevated WBC’s,
Q291 Neurology
A boy with worsening ear pair and headaches
recently had otitis media. Temp = 100.4F. The left
mastoid is tender and swollen. What is the next
step?

A) Antibiotics and follow up


B) MRI
Q291 Neurology
A boy with worsening ear pair and headaches
recently had otitis media. Temp = 100.4F. The left
mastoid is tender and swollen. What is the next
step?

A) Antibiotics and follow up


B) MRI
Q291 Neurology
A boy with worsening ear pair and headaches
recently had otitis media. Temp = 100.4F. The left
mastoid is tender and swollen. What is the next
step?

A) Antibiotics and follow up


B) MRI
Direct spread of bacteria from otitis media or mastoiditis can create an abscess in the
temporal lobe. Presentation can include headache, morning vomiting, fever, and focal
neurologic deficits. Diagnosis: MRI (or CT) to confirm; ring enhancing lesion. Tx:
antibiotics; surgery may be required to remove infected bone and drain the mastoid
Q292 Neurology
A patient with subacute symptoms of meningitis
has CSF findings of WBC's = 350, glucose = 25, and
protein = 180. What is the pathogen?

A) Neisseria
B) Tuberculosis
C) Herpes
Q292 Neurology
A patient with subacute symptoms of meningitis
has CSF findings of WBC's = 350, glucose = 25, and
protein = 180. What is the pathogen?

A) Neisseria
B) Tuberculosis
C) Herpes
Q292 Neurology
A patient with subacute symptoms of meningitis
has CSF findings of WBC's = 350, glucose = 25, and
protein = 180. What is the pathogen?

A) Neisseria - progresses within hours (not weeks) to septic shock and death
B) Tuberculosis*
*similar values to fungal meningitis (cryptococcus)

C) Herpes
Imaging shows basilar
meningeal enhancement
& hydrocephalus!
Q293 Neurology
An infant with hepatosplenomegaly, jaundice,
and periventricular calcifications most likely
has:

A) Hydrocephalus
B) Sensorineural hearing loss
Q293 Neurology
An infant with hepatosplenomegaly, jaundice,
and periventricular calcifications most likely
has:

A) Hydrocephalus
B) Sensorineural hearing loss
Q293 Neurology
An infant with hepatosplenomegaly, jaundice,
and periventricular calcifications most likely
has:

A) Hydrocephalus
B) Sensorineural hearing loss
CMV is the most common congenital infection (90%
asymptomatic); microcephaly & thrombocytopenia are also
common. The most common symptom is sensorineural
hearing loss (can develop later in childhood). Give
valganciclovir/ganciclovir to symptomatic kids!
Q294 Neurology
A 10-year-old boy has severe ear pain, unilateral
facial paralysis, and a vesicular rash in the
external auditory canal. What is the pathogen?

A) Varicella
B) Pneumococcus
Q294 Neurology
A 10-year-old boy has severe ear pain, unilateral
facial paralysis, and a vesicular rash in the
external auditory canal. What is the pathogen?

A) Varicella
B) Pneumococcus
Q294 Neurology
A 10-year-old boy has severe ear pain, unilateral
facial paralysis, and a vesicular rash in the
external auditory canal. What is the pathogen?

A) Varicella This is the triad of herpes zoster oticus ("Ramsay Hunt Syndrome");

B) Pneumococcus
pneumococcus can cause acute mastoiditis - severe ear
pain; no vesicular rash
Q295 Neurology
Pronator drift is a sign of:

A) impaired proprioception
B) UMN lesion
Q295 Neurology
Pronator drift is a sign of:

A) impaired proprioception
B) UMN lesion
Q295 Neurology
Pronator drift is a sign of:
Position sense (proprioception) is tested by holding the most distal joint of a digit by its sides
and moving it slightly up or down.
A) impaired proprioception
B) UMN lesion
Pronator drift is a sensitive and specific
sign for UMN (or corticospinal tract
disease) affecting the upper
extremities. Patient outstretches arms
with palms up and eyes closed - in
UMN lesions, the affected arms drifts
downward and palm pronates.
Q296 Neurology
A 41-year-old man presents with pharyngeal
muscle weakness, slurred speech, tongue
fasciculations, and a jaw jerk reflex. What is the
next step?

A) CK level
B) Electromyography
Q296 Neurology
A 41-year-old man presents with pharyngeal
muscle weakness, slurred speech, tongue
fasciculations, and a jaw jerk reflex. What is the
next step?

A) CK level
B) Electromyography
Q296 Neurology
A 41-year-old man presents with pharyngeal
muscle weakness, slurred speech, tongue
fasciculations, and a jaw jerk reflex. What is the
next step?

A) CK level
B) Electromyography
ALS presents with bulbar symptoms initially in 20% of cases;
electromyography, nerve conduction studies, and MRI to exclude other
diagnoses
Q297 Neurology
Livedo reticularis, mononeuritis multicomplex, and
elevated ESR are seen in:

A) Amyloidosis
B) Polyarteritis nodosa
Q297 Neurology
Livedo reticularis, mononeuritis multicomplex, and
elevated ESR are seen in:

A) Amyloidosis
B) Polyarteritis nodosa
Q297 Neurology
Livedo reticularis, mononeuritis multicomplex, and
elevated ESR are seen in:

A) Amyloidosis
B) Polyarteritis nodosa
Other findings: fever, weight loss, skin
nodules, purpura, ulcers, renal insufficiency,
seizures, headache, mesenteric
ischemia/infarction; intestinal angina (chest
pain with eating); myalgias, arthritis; NO lung
involvement
Q298 Neurology
A 34-year-old woman with no significant medical
history presents with unilateral upper and lower
facial drooping over several hours. Vital signs are
within normal limits; hearing is intact. What is the
next step?

A) MRI
B) Nothing
Q298 Neurology
A 34-year-old woman with no significant medical
history presents with unilateral upper and lower
facial drooping over several hours. Vital signs are
within normal limits; hearing is intact. What is the
next step?

A) MRI
B) Nothing
Q298 Neurology
A 34-year-old woman with no significant medical
history presents with unilateral upper and lower
facial drooping over several hours. Vital signs are
within normal limits; hearing is intact. What is the
next step? -there may be altered taste sensation
Facial Palsy

-tx: high dose steroids +/- acyclovir, and eye


protection (e.g., lubrication and taping)
A) MRI -MRI would only be done to exclude stroke as a
B) Nothing cause of facial palsy - but here she has no risks
and it occurred over hours (vs.
minutes/seconds)
Q299 Neurology
A 26-year-old woman presents with bilateral upper
extremity weakness and diffuse abdominal pain of
several days’ duration. Urobilinogen is positive.
What will reveal the diagnosis?

A) Genetic testing for JAK2


B) Urine porphyrin level
Q299 Neurology
A 26-year-old woman presents with bilateral upper
extremity weakness and diffuse abdominal pain of
several days' duration. Urobilinogen is positive.
What will reveal the diagnosis?

A) Genetic testing for JAK2


B) Urine porphyrin level
Q299 Neurology
A 26-year-old woman presents with bilateral upper
extremity weakness and diffuse abdominal pain of
several days' duration. Urobilinogen is positive.
What will reveal the diagnosis?

B) Urine porphyrin level


Acute intermittent porphyria: autosomal dominant; reduced activity of porphobilinogen
deaminase, buildup of porphobilinogen and ALA; present with neurotoxicity in various organ
systems: neuropathic abdominal pain, peripheral neuropathy (esp. proximal upper extremities),
autonomic dysfunction, and neuropsychiatric manifestations (e.g., restlessness, hallucinations)
Q300 Neurology
43-year-old man presents with muscles that
contract involuntarily, causing the head to twist or
turn to one side. Complete neuro exam is normal.
Treatment?

A) Risperidone
B) Botulinum toxin
Q300 Neurology
43-year-old man presents with muscles that
contract involuntarily, causing the head to twist or
turn to one side. Complete neuro exam is normal.
Treatment?

A) Risperidone
B) Botulinum toxin
Q300 Neurology
43-year-old man presents with muscles that
contract involuntarily, causing the head to twist or
turn to one side. Complete neuro exam is normal.
Treatment?

A) Risperidone – Tourette's requires at least one year


of motor and verbal ticks; presents in childhood (tetrabenazine preferred)

B) Botulinum toxin – for cervical


dystonia (more specific: spasmodic torticollis)
Q301 Neurology
An HIV patient has developed numbness and
pinpricks in his bilateral lower extremities; motor
strength is intact, but the ankle jerk reflex is
decreased bilaterally. What is the best treatment?

A) Prednisone
B) Gabapentin
Q301 Neurology
An HIV patient has developed numbness and
pinpricks in his bilateral lower extremities; motor
strength is intact, but the ankle jerk reflex is
decreased bilaterally. What is the best treatment?

A) Prednisone
B) Gabapentin
Q301 Neurology
An HIV patient has developed numbness and
pinpricks in his bilateral lower extremities; motor
strength is intact, but the ankle jerk reflex is
decreased bilaterally. What is the best treatment?
Distal symmetric polyneuropathy is the most common
A) Prednisone form of HIV neuropathy (unclear mechanism). Seen in low
CD4 counts. Insidious onset, numbness, tingling, burning in
B) Gabapentin bilateral toes/feet. Motor strength is intact. Gabapentin if
first line; SNRI's (e.g., venlafaxine) can also be used.
Note: Distal symmetric (Prednisone doesn't help and should be avoided in HIV
polyneuropathy can also be patients who are already immunosuppressed).
seen in chronic alcoholics
Q302 Neurology
A 34-year-old man has sudden onset dysphagia,
dysarthria, and blurred vision, followed by
decreased bilateral upper extremity muscle
strength. What is the next step?

A) Atropine
B) Methylprednisone
C) Antitoxin
Q302 Neurology
A 34-year-old man has sudden onset dysphagia,
dysarthria, and blurred vision, followed by
decreased bilateral upper extremity muscle
strength. What is the next step?

A) Atropine
B) Methylprednisone
C) Antitoxin
Q302 Neurology
A 34-year-old man has sudden onset dysphagia,
dysarthria, and blurred vision, followed by
decreased bilateral upper extremity muscle
strength. What is the next step?
(atropine is for cholinergic toxicity - bradycardia, vomiting, diarrhea; steroids
are for acute exacerbations of MS - neurologic deficits in space and time)

A) Atropine Symptoms are concerning for foodborne botulism (e.g.,

B) Methylprednisone
canned food contaminated with botulinum toxin); leads to
bilateral cranial neuropathies and symmetric descending

C) Antitoxin
muscle weakness; Can also present with diaphragmatic
weakness with respiratory failure
treat with antitoxin even before confirming diagnosis.
Q303 Neurology
A 30-year-old woman presents with a several
month history of fatigue, ptosis, decreased grip
strength, and inability to "let go" of her grip. She is
asked to close her eyes and has difficult time
opening them. What is this condition associated
with?

A) Post-synaptic acetylcholine receptor


B) Hypogonadism and cataracts
Q303 Neurology
A 30-year-old woman presents with a several
month history of fatigue, ptosis, decreased grip
strength, and inability to "let go" of her grip. She is
asked to close her eyes and has difficult time
opening them. What is this condition associated
with?

A) Post-synaptic acetylcholine receptor


B) Hypogonadism and cataracts
Q303 Neurology
A 30-year-old woman presents with a several
month history of fatigue, ptosis, decreased grip
strength, and inability to "let go" of her grip. She is
asked to close her eyes and has difficult time
opening them. What is this condition associated
with?

A) Post-synaptic acetylcholine receptor


B) Hypogonadism and cataracts Myotonic dystrophy
Q304 Neurology
A 67-year-old woman has symmetric
proximal limb weakness, reduced deep
tendon reflexes, and chronic cough. Chest
CT shows a lung mass. What is the
mechanism?

A) Muscle fiber injury


B) Autoantibodies against voltage-gated
calcium channels
Q304 Neurology
A 67-year-old woman has symmetric
proximal limb weakness, reduced deep
tendon reflexes, and chronic cough. Chest
CT shows a lung mass. What is the
mechanism?

A) Muscle fiber injury


B) Autoantibodies against voltage-gated
calcium channels
Q304 Neurology
A 67-year-old woman has symmetric
proximal limb weakness, reduced deep
tendon reflexes, and chronic cough. Chest
CT shows a lung mass. What is the
mechanism?
Lambert-Eaton syndrome is caused by autoantibodies directed
against calcium channels leading to proximal muscle weakness

A) Muscle fiber injury


and depressed deep tendon reflexes; frequently associated with
underlying malignancy (e.g., small cell lung cancer).

B) Autoantibodies against voltage-gated


calcium channels
Q305 Neurology
Which tremor decreases with voluntary
movement?

A) Essential
B) Parkinson
C) Cerebellar
D) Physiologic
Q305 Neurology
Which tremor decreases with voluntary
movement?

A) Essential
B) Parkinson
C) Cerebellar
D) Physiologic
Q305 Neurology
Which tremor decreases with voluntary
movement?
treat ET with propranolol! Then primidone
A) Essential – gets worse with movement, improves with alcohol, no other neurologic deficits;
B) Parkinson - asymmetric
C) Cerebellar – gets worse as you approach target
D) Physiologic – worse with movement, anxiety, and caffeine; not usually visible
Q306 Neurology
A businesswoman has weakness on foot and big
toe dorsiflexion and impaired sensation over the
lateral shin and dorsal foot. What is the
mechanism?

A) Her disc got herniated


B) She crosses her legs too much
Q306 Neurology
A businesswoman has weakness on foot and big
toe dorsiflexion and impaired sensation over the
lateral shin and dorsal foot. What is the
mechanism?

A) Her disc got herniated


B) She crosses her legs too much
Q306 Neurology
A businesswoman has weakness on foot and big
toe dorsiflexion and impaired sensation over the
lateral shin and dorsal foot. What is the
mechanism?

A) Her disc got herniated


B) She crosses her legs too much
she is presenting with common fibular nerve neuropathy (results
from damage or compression - e.g., prolonged crossing legs)
Q307 Neurology
A 28-year-old woman with Crohn disease has a
month history of worsening fatigue and painful
feet paresthesia. Reflexes are intact and
vibration/proprioception are intact. Hg = 8g/dL.
What is the next step?

A) Mercury level
B) MRI of spine
C) Serum B12 levels
Q307 Neurology
A 28-year-old woman with Crohn disease has a
month history of worsening fatigue and painful
feet paresthesia. Reflexes are intact and
vibration/proprioception are intact. Hg = 8g/dL.
What is the next step?

A) Mercury level
B) MRI of spine
C) Serum B12 levels
Q307 Neurology
A 28-year-old woman with Crohn disease has a
month history of worsening fatigue and painful
feet paresthesia. Reflexes are intact and
vibration/proprioception are intact. Hg = 8g/dL.
What is the next step?
Vitamin B12 deficiency can be seen in Crohn disease due to
reduced absorption of intrinsic factor-vitamin b12 complex in
the ileum. Vitamin B12 is required for myelin formation (leads
A) Mercury level to SCD - demyelination of the dorsal column - but initially may
present as lower extremity paresthesias with no other
B) MRI of spine neurologic findings); vitamin b12 is also needed for DNA
(megaloblastic anemia). (Mercury poisoning would involve
C) Serum B12 levels muscle weakening and areflexia)
Q308 Neurology
A 49-year-old woman with a history of HTN (controlled
with ACEi’s& hydrochlorothiazide) now has severe
weakness and some pain in her thigh muscles. Neuro
exam reveals hyporeflexia and decreased strength in all
muscle groups. EKG shows broad flat T waves and U
waves. What is the diagnosis?

A) Hemorrhage
B) MS
C) Electrolyte abnormality
Q308 Neurology
A 49-year-old woman with a history of HTN (controlled
with ACEi’s& hydrochlorothiazide) now has severe
weakness and some pain in her thigh muscles. Neuro
exam reveals hyporeflexia and decreased strength in all
muscle groups. EKG shows broad flat T waves and U
waves. What is the diagnosis?

A) Hemorrhage
B) MS
C) Electrolyte abnormality
Q308 Neurology
A 49-year-old woman with a history of HTN (controlled
with ACEi’s& hydrochlorothiazide) now has severe
weakness and some pain in her thigh muscles. Neuro
exam reveals hyporeflexia and decreased strength in all
muscle groups. EKG shows broad flat T waves and U
waves. What is the diagnosis?
Hypokalemia (can be caused by thiazides) can cause
A) Hemorrhage weakness, fatigue, and muscle cramps; severe
hypokalemia can lead to paralysis; EKG may show flat and
B) MS broad T waves and U waves; treat with potassium.
C) Electrolyte abnormality
Q309 Neurology
A man presents with numbness in the medial hand over
the last several months. He also has decreased sensation
over the 4th and 5th digits along with decreased grip
strength and wrist flexion. Where is the nerve injury?

A) Ulnar nerve at the wrist


B) Ulnar nerve at the elbow
Q309 Neurology
A man presents with numbness in the medial hand over
the last several months. He also has decreased sensation
over the 4th and 5th digits along with decreased grip
strength and wrist flexion. Where is the nerve injury?

A) Ulnar nerve at the wrist


B) Ulnar nerve at the elbow
Q309 Neurology
A man presents with numbness in the medial hand over
the last several months. He also has decreased sensation
over the 4th and 5th digits along with decreased grip
strength and wrist flexion. Where is the nerve injury?

A) Ulnar nerve at the wrist


B) Ulnar nerve at the elbow
injury at wrist can lead to numbness and paresthesia in
medial 1.5. fingers along with intrinsic hand weakness;
injury at elbow has additional hypothenar eminence
numbness and decreased grip strength (flexor digitorum
profundus)/wrist flexion (flexor carpi ulnaris).
Q310 Neurology
A 41-year-old man presents with several
weeks of low-grade fevers, fatigue, and
dry eyes and now this morning woke up
with facial nerve palsy. Calcium = 11 mg/dL.
What is the cause of the palsy?

A) Idiopathic
B) Sarcoidosis
Q310 Neurology
A 41-year-old man presents with several
weeks of low-grade fevers, fatigue, and
dry eyes and now this morning woke up
with facial nerve palsy. Calcium = 11 mg/dL.
What is the cause of the palsy?

A) Idiopathic
B) Sarcoidosis
Q310 Neurology
A 41-year-old man presents with several
weeks of low-grade fevers, fatigue, and
dry eyes and now this morning woke up
with facial nerve palsy. Calcium = 11 mg/dL.
What is the cause of the palsy?
Facial nerve palsy is a manifestation of sarcoidosis, as well as

A) Idiopathic
hypercalcemia, systemic symptoms (fevers, fatigue), and dry eyes
(keratoconjunctivitis sicca); lung symptoms are absent in half of
B) Sarcoidosis patients. Other symptoms of sarcoidosis: erythema nodosum,
uveitis, diabetes insipidus, AV block, cardiomyopathy, hepatomegaly,
LFT abnormalities, parotid gland swelling, & polyarthritis
Q311 Neurology
A man has deficits with knee
extension after an accident, as
well as reduced sensation in the
anterior thigh and medial lower
leg. What nerve was damaged?

A) Common fibular
B) Femoral
Q311 Neurology
A man has deficits with knee
extension after an accident, as
well as reduced sensation in the
anterior thigh and medial lower
leg. What nerve was damaged?

A) Common fibular
B) Femoral
Q311 Neurology
A man has deficits with knee
extension after an accident, as
well as reduced sensation in the
anterior thigh and medial lower
leg. What nerve was damaged?

A) Common fibular
B) Femoral (L2-L4)
The femoral nerve can get injured from hip dislocation,
pelvic fracture, or hematoma in the thigh
Q312 Neurology
Skin depigmentation, ataxia with positive
Romberg sign, anemia, and brittle hair are
seen in which vitamin deficiency:

A) Iron
B) Selenium
C) Zinc
D) Copper
Q312 Neurology
Skin depigmentation, ataxia with positive
Romberg sign, anemia, and brittle hair are
seen in which vitamin deficiency:

A) Iron
B) Selenium
C) Zinc
D) Copper
Q312 Neurology
Skin depigmentation, ataxia with positive
Romberg sign, anemia, and brittle hair are
seen in which vitamin deficiency:

A) Iron Copper deficiency (acquired) is usually


due to malabsorption from prior gastric
B) Selenium surgery. Can be seen in malabsorptive

C) Zinc
diseases (e.g., celiac); also, in excessive
ingestion of zinc, competes with copper
D) Copper for absorption. Neurologic deficits
similar to vitamin B12 deficiency. Tx:
copper, stop zinc
Q313 Neurology
A 34-year-old male IV drug user is found
with weakness, lethargy, ptosis, dilated
pupils, acute hypoxemic respiratory failure,
and several cutaneous abscesses. RR = 19.
What is the next step?

A) Drug screen
B) Botulinum antitoxin
Q313 Neurology
A 34-year-old male IV drug user is found
with weakness, lethargy, ptosis, dilated
pupils, acute hypoxemic respiratory failure,
and several cutaneous abscesses. RR = 19.
What is the next step?

A) Drug screen
B) Botulinum antitoxin
Q313 Neurology
A 34-year-old male IV drug user is found
with weakness, lethargy, ptosis, dilated
pupils, acute hypoxemic respiratory failure,
and several cutaneous abscesses. RR = 19.
What is the next step? cases due to puncture wound contamination (e.g.,
Symptoms suggest C botulinum infection; most

IV drugs); also when spores (infant botulism) or

A) Drug screen
preformed toxin (foodborne botulism) are ingested.
Urgent treatment with antitoxin given even before

B) Botulinum antitoxin diagnostic identification (e.g., culture). (Not mere


opioid intoxication here which can cause weakness,
lethargy, respiratory failure; that would cause
bradypnea; ptosis is uncommon)
Q314 Neurology
An 8-year-old boy has excessive daytime
sleepiness despite good sleep hygiene. He
has difficulty opening his hands after
squeezing the physician's fingers. What is
the mode of inheritance?

A) Autosomal recessive
B) Autosomal dominant
Q314 Neurology
An 8-year-old boy has excessive daytime
sleepiness despite good sleep hygiene. He
has difficulty opening his hands after
squeezing the physician's fingers. What is
the mode of inheritance?

A) Autosomal recessive
B) Autosomal dominant
Q314 Neurology
An 8-year-old boy has excessive daytime
sleepiness despite good sleep hygiene. He
has difficulty opening his hands after
squeezing the physician's fingers. What is
the mode of inheritance?
Myotonic dystrophy - CTG repeat expansion
on DMPK gene; often presents in
A) Autosomal recessive adolescence/adulthood but can present in
childhood - can manifest with excessive
B) Autosomal dominant daytime sleepiness. Grip myotonia is a
hallmark finding. Dysphagia and arrythmias
may also be seen.
Q315 Neurology
A 12-year-old girl has a 2-day history of rapid
onset bilateral abnormal upper extremity and
then lower extremity movements during the
day. Sensation is normal, neuro exam is
otherwise normal, and mental status is intact.
What is the next best step?

A) Psychiatric assessment
B) Antistreptolysin O titer
Q315 Neurology
A 12-year-old girl has a 2-day history of rapid
onset bilateral abnormal upper extremity and
then lower extremity movements during the
day. Sensation is normal, neuro exam is
otherwise normal, and mental status is intact.
What is the next best step?

A) Psychiatric assessment
B) Antistreptolysin O titer
Q315 Neurology
A 12-year-old girl has a 2-day history of rapid
onset bilateral abnormal upper extremity and
then lower extremity movements during the
day. Sensation is normal, neuro exam is
otherwise normal, and mental status is intact.
What is the next best step? due to abnormal basal ganglia signaling); in
chorea (involuntary contractions of muscles

kids, often due to GAS (Sydenham chorea)


that can also be a manifestation of acute RF;

A) Psychiatric assessment it often self-resolves within months, but


recurrence is common; get ASO; also, test

B) Antistreptolysin O titer with EKG and echo for rheumatic fever


Q316 Neurology
Which symptom associated with facial nerve palsy is
considered a "red flag" requiring further workup?

A) Sparing of the upper face


B) Flu like illness
C) Hearing deficit
D) Lower limb weakness
E) All of the above
Q316 Neurology
Which symptom associated with facial nerve palsy is
considered a "red flag" requiring further workup?

A) Sparing of the upper face


B) Flu like illness
C) Hearing deficit
D) Lower limb weakness
E) All of the above
Q316 Neurology
Which symptom associated with facial nerve palsy is
considered a "red flag" requiring further workup?

A) Sparing of the upper face - brain MRI to r/o stroke


B) Flu like illness - Lyme serology
C) Hearing deficit - MRI skull base to r/o CPA tumor
D) Lower limb weakness - lumbar puncture to r/o GBS
E) All of the above
Q317 Neurology
A 13-year-old girl presents with a worsening headache
over the last 24 hours (w/ nausea and vomiting) - the
headache is in the right frontal area and is pulsatile.
Similar headaches have occurred monthly. Neuro exam/VS
are normal. What is the next best diagnostic step?

A) CT scan of the head


B) Lumbar puncture
C) No testing
Q317 Neurology
A 13-year-old girl presents with a worsening headache
over the last 24 hours (w/ nausea and vomiting) - the
headache is in the right frontal area and is pulsatile.
Similar headaches have occurred monthly. Neuro exam/VS
are normal. What is the next best diagnostic step?

A) CT scan of the head


B) Lumbar puncture
C) No testing
Q317 Neurology
A 13-year-old girl presents with a worsening headache
over the last 24 hours (w/ nausea and vomiting) - the
headache is in the right frontal area and is pulsatile.
Similar headaches have occurred monthly. Neuro exam/VS
are normal. What is the next best diagnostic step?
with migraine; triggers can include sleep deprivation, stress, and
menses; migraine headaches can persist up to 3 days; loud
A) CT scan of the head noises typically exacerbate symptoms; diagnosis is clinical; no
neuroimaging is necessary (unless we saw weakness, ataxia,
B) Lumbar puncture etc.); tx migraines with Tylenol/NSAIDs; refractory, treat with
triptan (a serotonin receptor agonist to vasoconstrict cranial
C) No testing arteries)
Q318 Neurology
Progressive headache, neck pain/stiffness,
obesity, along with elevated ICP despite normal
CSF content:

A) idiopathic ICH
B) meningitis
Q318 Neurology
Progressive headache, neck pain/stiffness,
obesity, along with elevated ICP despite normal
CSF content:

A) idiopathic ICH
B) meningitis
Q318 Neurology
Progressive headache, neck pain/stiffness,
obesity, along with elevated ICP despite normal
CSF content:
MRI in idiopathic ICH
Headaches with ICP classically worsen is typically normal –
while patient is in recumbent position only done to exclude
secondary causes of
(e.g., during night or waking up) or is elevated ICP; if

A) idiopathic ICH straining (e.g., bowel movements), but


atypical patterns exist. Neck pain and
negative, perform
lumbar puncture to

B) meningitis
diagnose idiopathic
stiffness may also be present, ICH;
-tx: weight loss,
especially in young obese women. Also acetazolamide, if not,
Papilledema can cause transient seen in patients who take isotretinoin shunting (important
vision loss, changes in head to treat or it can lead
positioning (e.g., when standing or tetracyclines (e.g., for severe acne). to gradual permanent
up). can cause enlargement of the LP shows elevated opening pressure vision loss)
physiologic blind spots.
(>250).
Q319 Neurology
A 45-year-old woman presents with a month
history of constant pressure headaches in the
temporal and occipital regions. Episodes last an
hour and often occur at work. She has no nausea
or visual symptoms. Neuro exam is normal. What
is the diagnosis?

A) Migraine without aura


B) Tension headache
Q319 Neurology
A 45-year-old woman presents with a month
history of constant pressure headaches in the
temporal and occipital regions. Episodes last an
hour and often occur at work. She has no nausea
or visual symptoms. Neuro exam is normal. What
is the diagnosis?

A) Migraine without aura


B) Tension headache
Q319 Neurology
A 45-year-old woman presents with a month
history of constant pressure headaches in the
temporal and occipital regions. Episodes last an
hour and often occur at work. She has no nausea
or visual symptoms. Neuro exam is normal. What
is the diagnosis?

A) Migraine without aura


B) Tension headache minutes-7 days; tenderness in head, neck, shoulders
Tension headaches - related to stress; dull & tight; last 30
Q320 Neurology
A 40-year-old lady presents with achy jaw pain,
facial pain, and headaches. What is the most
likely diagnosis?

A) Giant cell arteritis


B) TMJ
Q320 Neurology
A 40-year-old lady presents with achy jaw pain,
facial pain, and headaches. What is the most
likely diagnosis?

A) Giant cell arteritis


B) TMJ
Q320 Neurology
A 40-year-old lady presents with achy jaw pain,
facial pain, and headaches. What is the most
likely diagnosis?

A) Giant cell arteritis


B) TMJ
Giant cell arteritis is
extremely rare in people
under age of 50!!
Q321 Neurology
Which of the following has NOT shown to be
helpful in preventing migraines?

A) Propranolol
B) Amitriptyline
C) Sertraline
D) Topiramate
Q321 Neurology
Which of the following has NOT shown to be
helpful in preventing migraines?

A) Propranolol
B) Amitriptyline
C) Sertraline
D) Topiramate
Q321 Neurology
Which of the following has NOT shown to be
helpful in preventing migraines?

A) Propranolol – esp. in pregnancy


B) Amitriptyline (& venlafaxine)
C) Sertraline – SSRI’s not helpful
D) Topiramate
Triptans, NSAIDs, and
ergotamines are abortive
therapies!
Q322 Neurology
Which of the following are contraindicated in
migraine with aura?

A) combined estrogen/progestin pill


B) vaginal ring
C) estradiol transdermal patch
D) all of the above
Q322 Neurology
Which of the following are contraindicated in
migraine with aura?

A) combined estrogen/progestin pill


B) vaginal ring
C) estradiol transdermal patch
D) all of the above
Q322 Neurology
Which of the following are contraindicated in
migraine with aura?

A) combined estrogen/progestin pill


B) vaginal ring
C) estradiol transdermal patch
D) all of the above
Migraine with aura patients should avoid estrogen
containing contraceptives due to risk of thrombosis leading
to ischemic stroke
Q323 Neurology
Headaches and seizures in a post-partum woman
with normal blood pressure and a family history
of DVT. Next step?

A) Magnesium sulfate
B) MRI of head with MR venography
Q323 Neurology
Headaches and seizures in a post-partum woman
with normal blood pressure and a family history
of DVT. Next step?

A) Magnesium sulfate
B) MRI of head with MR venography
Q323 Neurology
Headaches and seizures in a post-partum woman
with normal blood pressure and a family history
of DVT. Next step?

A) Magnesium sulfate
B) MRI of head with MR venography
Suspect cerebral vein thrombosis (highly variable
presentations depending on location - headache, elevated
ICP, seizures, encephalopathy); MRI with MR venography is
very sensitive; treat with anticoagulation (heparin or
warfarin)
Q324 Neurology
A 12-year-old girl with DM has a 2-week history
of blurry vision, and sometimes has nausea and
headaches. Visual fields have enlarged blind
spots bilaterally. Retinal exam is shown. What is
the diagnosis?

A) Retinitis pigmentosa
B) Idiopathic ICH
Q324 Neurology
A 12-year-old girl with DM has a 2-week history
of blurry vision, and sometimes has nausea and
headaches. Visual fields have enlarged blind
spots bilaterally. Retinal exam is shown. What is
the diagnosis?

A) Retinitis pigmentosa
B) Idiopathic ICH
Q324 Neurology
-Idiopathic ICH, commonly in young obese women but can be found
even in nonobese children; ICP causes optic disc swelling (on exam
shows blurred margins); vessels may be engorged/leaky ("splinter
hemorrhages"). In children, increased ICP often affects the abducens
nerve (diplopia, impaired eye abduction); dx: neuroimaging, if not, LP is
diagnostic; treatment - acetazolamide

A) Retinitis pigmentosa
B) Idiopathic ICH
Q325 Neurology
A 10-year-old girl is brought to the ED for visual
disturbances. She saw "flashing lights" and then spots of
vision loss; this fully resolves a half hour later. Since then
she has had a bifrontal throbbing headache with nausea
(similar to two episodes in the last year). Vital signs and
physical exam is normal. What is the next step?

A) Intraocular pressure measurement


B) NSAIDs
Q325 Neurology
A 10-year-old girl is brought to the ED for visual
disturbances. She saw "flashing lights" and then spots of
vision loss; this fully resolves a half hour later. Since then
she has had a bifrontal throbbing headache with nausea
(similar to two episodes in the last year). Vital signs and
physical exam is normal. What is the next step?

A) Intraocular pressure measurement


B) NSAIDs
Q325 Neurology
A 10-year-old girl is brought to the ED for visual disturbances.
She saw "flashing lights" and then spots of vision loss; this
fully resolves a half hour later. Since then she has had a
bifrontal throbbing headache with nausea (similar to two
episodes in the last year). Vital signs and physical exam is
normal. What is the next step?
Closed angle glaucoma would cause severe eye pain and conjunctival redness
A) Intraocular pressure measurement
B) NSAIDs
likely has a migraine headache - pain is typically "throbbing" or "pulsatile"; in kids it is often
bifrontal. Nausea, vomiting, photophobia and phonophobia are seen. Visual auras are the
most common type of aura - bright, flashing lights (typically less than 1 hour).
Q326 Neurology
A 75-year-old man is brought in by his daughter due to
poor adherence to his diabetic regimen. She also
describes that he has been falling recently and has been
wetting his pants. Neuro exam shows absence of bilateral
ankle reflexes; PE shows a wide based gait. What is the
next step?

A) Place patient in nursing home


B) Get an MRI of the brain
Q326 Neurology
A 75-year-old man is brought in by his daughter due to
poor adherence to his diabetic regimen. She also
describes that he has been falling recently and has been
wetting his pants. Neuro exam shows absence of bilateral
ankle reflexes; PE shows a wide based gait. What is the
next step?

A) Place patient in nursing home


B) Get an MRI of the brain
Q326 Neurology
A 75-year-old man is brought in by his daughter due to
poor adherence to his diabetic regimen. She also
describes that he has been falling recently and has been
wetting his pants. Neuro exam shows absence of bilateral
ankle reflexes; PE shows a wide based gait. What is the
next step? Normal pressure hydrocephalus = cognitive impairment, urinary incontinence,
and gait abnormalities (wide based); a potentially reversible cause of dementia
(diagnosis: MRI, CT showings ventriculomegaly out of proportion to the sulci;
confirm with LP; treatment: ventriculoperitoneal shunt). [UMN signs may occur in
advanced disease, but here it is probably just related to age related changes].
B) Get an MRI of the brain
Q327 Neurology
A 66-year-old woman has a 2-year history of memory
loss, periodic confusion, fluctuations in cognition, visual
hallucinations, and parkinsonism. MRI shows generalized
cortical atrophy. What is the diagnosis?

A) Alzheimer
B) Parkinson
C) Lewy body dementia
Q327 Neurology
A 66-year-old woman has a 2-year history of memory
loss, periodic confusion, fluctuations in cognition, visual
hallucinations, and parkinsonism. MRI shows generalized
cortical atrophy. What is the diagnosis?

A) Alzheimer
B) Parkinson
C) Lewy body dementia
Q327 Neurology
A 66-year-old woman has a 2-year history of memory
loss, periodic confusion, fluctuations in cognition, visual
hallucinations, and parkinsonism. MRI shows generalized
cortical atrophy. What is the diagnosis?

A) Alzheimer – memory loss is early and prominent


B) Parkinson – dementia occurs much later on
confusion + hallucinations + parkinsonian motor symptoms (autopsy
C) Lewy body dementia shows eosinophilic intracytoplasmic inclusions - represent alpha
synuclein protein); tx: carbidopa-levodopa for parkinsonism and
cholinesterase inhibitors for cognitive impairment (second line - 2ns
generation antipsychotic)
Q328 Neurology
A 66-year-old man with no significant
medical history or medications has a 1-
year-history of left-hand tremor and
stiffness. He also has poor balance and
slowness. What is required for diagnosis?

A) CT scan
B) MRI
C) nothing
Q328 Neurology
A 66-year-old man with no significant
medical history or medications has a 1-
year-history of left-hand tremor and
stiffness. He also has poor balance and
slowness. What is required for diagnosis?

A) CT scan
B) MRI
C) nothing
Q328 Neurology
A 66-year-old man with no significant
medical history or medications has a 1-
year-history of left-hand tremor and
stiffness. He also has poor balance and
slowness. What is required for diagnosis?

A) CT scan Parkinson disease is a clinical diagnosis - required at least 2 of the

B) MRI
following: rest tremor, rigidity, and bradykinesia; no imaging is
required for diagnosis (CT scan is often unremarkable; MRI may be
C) nothing helpful to exclude stroke or NPH - but these can often be ruled out
with a good history)
Q329 Neurology
A 66-year-old man has had slowly progressive right-
hand weakness for 3 months. Tendon reflexes are 3+ in
the right arm and 1+ in the left; electrophysiologic
studies show widespread fibrillations and positive sharp
waves in all extremity muscle groups. Where is the
abnormality?

A) Peripheral nerve
B) Anterior horn cell
Q329 Neurology
A 66-year-old man has had slowly progressive right-
hand weakness for 3 months. Tendon reflexes are 3+ in
the right arm and 1+ in the left; electrophysiologic
studies show widespread fibrillations and positive sharp
waves in all extremity muscle groups. Where is the
abnormality?

A) Peripheral nerve
B) Anterior horn cell
Q329 Neurology
A 66-year-old man has had slowly progressive right-
hand weakness for 3 months. Tendon reflexes are 3+ in
the right arm and 1+ in the left; electrophysiologic
studies show widespread fibrillations and positive sharp
waves in all extremity muscle groups. Where is the
abnormality?
ALS leads to degeneration of cells in the anterior horn of the spinal cord;
asymmetric weakness and UMN/LMN signs
A) Peripheral nerve
B) Anterior horn cell
Q330 Neurology
A 66-year-old woman with a 1-year history of
compulsions, executive dysfunction, and apathy
tried to hug and kiss men on the street. What
should be given?

A) Donepezil
B) Citalopram
Q330 Neurology
A 66-year-old woman with a 1-year history of
compulsions, executive dysfunction, and apathy
tried to hug and kiss men on the street. What
should be given?

A) Donepezil
B) Citalopram
Q330 Neurology
A 66-year-old woman with a 1-year history of
compulsions, executive dysfunction, and apathy
tried to hug and kiss men on the street. What
should be given?

A) Donepezil Acetylcholinesterase inhibitor for Alzheimer's


B) Citalopram
Frontotemporal dementia (Pick disease) is a common cause of early onset dementia (50's-60's).
Requires 3/6> of the following: disinhibition (e.g., sexual), apathy, loss of empathy, compulsive
behavior/rituals, hyperorality (e.g., smoking) and executive decline. First line management is with
behavioral interventions; if prominent sexual symptoms, treat with SSRI's or trazodone - decreased
sexual desire.
Q331 Neurology
A tremor that decreases with voluntary
movement (e.g., reaching for a remote) is seen in:

A) essential tremor
B) cerebellar tremor
C) basal ganglia dysfunction
Q331 Neurology
A tremor that decreases with voluntary
movement (e.g., reaching for a remote) is seen in:

A) essential tremor
B) cerebellar tremor
C) basal ganglia dysfunction
Q331 Neurology
A tremor that decreases with voluntary
movement (e.g., reaching for a remote) is seen in:

A) essential tremor
B) cerebellar tremor
C) basal ganglia dysfunction
Parkinson disease is due to a progressive loss of
dopaminergic neurons in the substantia nigra of the basal
ganglia; this leads to interrupted motor pathways
between the basal ganglia, thalamus, and motor cortex.
Q332 Neurology
Cortical and subcortical infarctions are seen in:

A) Lewy body dementia


B) Vascular dementia
C) NPH
Q332 Neurology
Cortical and subcortical infarctions are seen in:

A) Lewy body dementia


B) Vascular dementia
C) NPH
Q332 Neurology
Cortical and subcortical infarctions are seen in:

A) Lewy body dementia


B) Vascular dementia
C) NPH Ventriculomegaly; ataxia

Vascular dementia - sudden or stepwise decline in


executive function with some memory loss. are
focal neurologic deficits, and neuroimaging
classically demonstrates multiple small infarctions
- cortical/sub-cortical.
Q333 Neurology
In which lobe does Alzheimer's disease begin?

A) Frontal
B) Parietal
C) Temporal
Q333 Neurology
In which lobe does Alzheimer's disease begin?

A) Frontal
B) Parietal
C) Temporal
Q333 Neurology
In which lobe does Alzheimer's disease begin?

A) Frontal – Pick disease


B) Parietal - primary progressive aphasia
C) Temporal
temporal lobe atrophy on neuroimaging, most
prominent in the hippocampi, surrounding
medial temporal lobes.
Cholinesterase inhibitors such as donepezil, rivastigmine, and
galantamine (i.e., they increase acetylcholine in the BRAIN) may
provide some symptomatic relief of cognitive symptoms and
temporarily improve functioning in Alzheimer disease; however,
don’t’t alter the disease course.
Q334 Neurology
A 55-year-old man has rapidly
progressive dementia and startle-
induced involuntary muscle spasms.
EEG shows periodic sharp-wave
complexes. What is the outcome?

A) Death within a year


B) A decade more of life with proper
medication
Q334 Neurology
A 55-year-old man has rapidly
progressive dementia and startle-
induced involuntary muscle spasms.
EEG shows periodic sharp-wave
complexes. What is the outcome?

A) Death within a year


B) A decade more of life with proper
medication
Q334 Neurology
A 55-year-old man has rapidly
progressive dementia and startle-
induced involuntary muscle spasms.
EEG shows periodic sharp-wave
complexes. What is the outcome?
Creutzfeldt-Jakob disease - a fatal prion disease; rapidly progressive dementia, myoclonus, mood
changes, and sleep disturbances; no treatment, death within a year

A) Death within a year


B) A decade more of life with proper
medication
Q335 Neurology
Caudate nucleus and putamen
atrophy result in degeneration of
neurons which produce which cells:

A) Glutamate
B) GABA
e

Q335 Neurology
a
;
p
r
e

Caudate nucleus and putamen


f
e
r

atrophy result in degeneration of


e
n
t

neurons which produce which cells:


i
a
l
d
e
g
e
n
e
r
A) Glutamate
B) GABA
a
t
i
o
n
o
f
G
A
B
A
p
r
o
d
Q335 Neurology
Caudate nucleus and putamen
atrophy result in degeneration of
neurons which produce which cells:

A) Glutamate
B) GABA
Huntington - autosomal dominant;
CAG nucleotide repeat; cognitive
impairment, chorea; preferential
degeneration of GABA producing
neurons in the caudate and putamen
Q336 Neurology
A heavy chronic alcoholic has a wide based gait
and needs a walker to ambulate. Cognition is
intact. Where is the defect?

A) Frontal lobe
B) Purkinje fibers
Q336 Neurology
A heavy chronic alcoholic has a wide based gait
and needs a walker to ambulate. Cognition is
intact. Where is the defect?

A) Frontal lobe
B) Purkinje fibers
Q336 Neurology
A heavy chronic alcoholic has a wide based gait
and needs a walker to ambulate. Cognition is
intact. Where is the defect?

A) Frontal lobe
B) Purkinje fibers
Alcoholic cerebellar degeneration
Q337 Neurology
Before a definitive diagnosis of Alzheimer’s is
made, what is an important lab value to obtain
in addition to CBC and CMP?

A) Folate level
B) Vitamin D level
C) Vitamin B12 level
Q337 Neurology
Before a definitive diagnosis of Alzheimer’s is
made, what is an important lab value to obtain
in addition to CBC and CMP?

A) Folate level
B) Vitamin D level
C) Vitamin B12 level
Q337 Neurology
Before a definitive diagnosis of Alzheimer’s is
made, what is an important lab value to obtain
in addition to CBC and CMP?

A) Folate level
B) Vitamin D level
C) Vitamin B12 level
Evaluation of Alzheimer's involves neuropsychological
testing, laboratory testing (complete blood count,
vitamin B12, TSH, complete metabolic panel), and
neuroimaging.
Q338 Neurology
Acute angle closure glaucoma can be
precipitated by:

A) Selegeline
B) Propranolol
C) Trihexyphenidyl
Q338 Neurology
Acute angle closure glaucoma can be
precipitated by:

A) Selegeline
B) Propranolol
C) Trihexyphenidyl
Q338 Neurology
Acute angle closure glaucoma can be
precipitated by:
Can also be

A) Selegeline
instigated by
walking into a
B) Propranolol dark room

C) Trihexyphenidyl
Anticholinergics (e.g., trihexyphenidyl for
Parkinson symptoms) can cause mydriasis and so
they can precipitate acute ACG (can result in
permanent vision loss within 2-5 hours of onset)
Q339 Neurology
A 64-year-old man who used to sing now has a
soft voice; he also coughs when he drinks; he
also has decreased smell. His right arm has
increased resistance to passive movement and
his posture is slow and stooped. What is the
diagnosis?

A) ALS
B) Parkinson
Q339 Neurology
A 64-year-old man who used to sing now has a
soft voice; he also coughs when he drinks; he
also has decreased smell. His right arm has
increased resistance to passive movement and
his posture is slow and stooped. What is the
diagnosis?

A) ALS
B) Parkinson
Q339 Neurology
A 64-year-old man who used to sing now has a
soft voice; he also coughs when he drinks; he
also has decreased smell. His right arm has
increased resistance to passive movement and
his posture is slow and stooped. What is the
diagnosis? Parkinson disease most often presents with tremor, but can also
present with bulbar symptoms: hypophonia, aspiration of
liquids, loss of olfactory function (change of taste); this is in
addition to upper limb rigidity and stooped posture/slow gait.

A) ALS (ALS can present with bulbar symptoms as well, but exam would
show UMN and LMN signs - atrophy, fasciculations,

B) Parkinson hyperreflexia) - but rigidity, loss of olfactory function, and


stooped posture are more seen in Parkinson
Q340 Neurology
A 65-year-old man presents with progressively
worsening concentration and judgement for the last 2
months. Nystagmus is present, along with mild
spasticity, myoclonus, and hyperreflexia. Gait is
unsteady. MRI shows cerebral atrophy and diffuse
bright signals in cortical white matter and portions of
the basal ganglia. What is the diagnosis?

A) Thiamine deficiency
B) Prion disease
Q340 Neurology
A 65-year-old man presents with progressively
worsening concentration and judgement for the last 2
months. Nystagmus is present, along with mild
spasticity, myoclonus, and hyperreflexia. Gait is
unsteady. MRI shows cerebral atrophy and diffuse
bright signals in cortical white matter and portions of
the basal ganglia. What is the diagnosis?

A) Thiamine deficiency
B) Prion disease
Q340 Neurology
A 65-year-old man presents with progressively
worsening concentration and judgement for the last 2
months. Nystagmus is present, along with mild
spasticity, myoclonus, and hyperreflexia. Gait is
unsteady. MRI shows cerebral atrophy and diffuse
bright signals in cortical white matter and portions of
the basal ganglia. What is the diagnosis?

A) Thiamine deficiency
B) Prion disease provoked by startle) and hyperreflexia seen in prion disease. Also MRI
Nystagmus and gait ataxis seen in both, but myoclonus (classically

more consistent with prion disease; in thiamine deficiency, MRI


classically reveals mamillary body atrophy and periaqueductal grey
matter hyperintensity.
Q341 Neurology
Initial diagnostic workup of a first-time
seizure in an adult includes:

A) serum electrolytes, glucose, urine


toxicology
B) no workup
Q341 Neurology
Initial diagnostic workup of a first-time
seizure in an adult includes:

A) serum electrolytes, glucose, urine


toxicology
B) no workup
Q341 Neurology
Initial diagnostic workup of a first-time
seizure in an adult includes:

A) serum electrolytes, glucose, urine


toxicology First time seizure in adults may be due to
B) no workup metabolic causes (e.g., hypoglycemia, electrolyte
disturbances) or toxic causes (e.g., amphetamine
use, benzo/alcohol withdrawal)
Q342 Neurology
A patient has generalized convulsive status
epilepticus and is given benzodiazepines; he
remains unresponsive. What is the next
step?

A) EEG
B) MRI
Q342 Neurology
A patient has generalized convulsive status
epilepticus and is given benzodiazepines; he
remains unresponsive. What is the next
step?

A) EEG
B) MRI
Q342 Neurology
A patient has generalized convulsive status
epilepticus and is given benzodiazepines; he
remains unresponsive. What is the next
step?

A) EEG Patients who remain unresponsive


after generalized convulsive status
B) MRI epilepticus require EEG to
differentiate between sedation due
to benzos and persistent
nonconvulsive status epilepticus.
Q343 Neurology
A 9-year-old boy stares into space and then
tilts his head to the right and develops right
arm twitching; the episode lasts 3 minutes;
the patient seemed tired and confused
afterward. What is the most likely
diagnosis?

A) Focal seizure
B) Absence seizure
Q343 Neurology
A 9-year-old boy stares into space and then
tilts his head to the right and develops right
arm twitching; the episode lasts 3 minutes;
the patient seemed tired and confused
afterward. What is the most likely
diagnosis?

A) Focal seizure
B) Absence seizure
Q343 Neurology
A 9-year-old boy stares into space and then
tilts his head to the right and develops right
arm twitching; the episode lasts 3 minutes;
the patient seemed tired and confused
afterward. What is the most likely
diagnosis?
Focal seizures originate in a single
hemisphere and can spread to both
A) Focal seizure hemispheres (episodes could last a few
B) Absence seizure minutes vs. absence seizure, about 10
seconds, no postictal period)
Q344 Neurology
A 20-year-old male noncompliant with
seizure meds develops status epilepticus for
10 minutes. What is he at risk for?

A) Cerebellar atrophy
B) Cortical necrosis
Q344 Neurology
A 20-year-old male noncompliant with
seizure meds develops status epilepticus for
10 minutes. What is he at risk for?

A) Cerebellar atrophy
B) Cortical necrosis
Q344 Neurology
A 20-year-old male noncompliant with
seizure meds develops status epilepticus for
10 minutes. What is he at risk for?

A) Cerebellar atrophy
B) Cortical necrosis
A brain with seizures > 5 minutes are at
increased risk of permanent cytotoxic
injury (cortical laminar necrosis is the
hallmark).
Q345 Neurology
A woman on phenytoin is seizure free for
several years and now wants to get
pregnant. What should she do?

A) Switch to carbamazepine
B) Stop phenytoin right away
C) Taper off phenytoin slowly
Q345 Neurology
A woman on phenytoin is seizure free for
several years and now wants to get
pregnant. What should she do?

A) Switch to carbamazepine
B) Stop phenytoin right away
C) Taper off phenytoin slowly
Q345 Neurology
A woman on phenytoin is seizure free for
several years and now wants to get
pregnant. What should she do?
(Carbamazepine is teratogenic; if she still needs seizure medication,
she can take lamotrigine or levetiracetam).
A) Switch to carbamazepine
B) Stop phenytoin right away
C) Taper off phenytoin slowly
In 60% of patients, epilepsy completes resolves; so a patient can stop
medication after 2 years seizure free; anti-epileptics need to be slowly
tapered.
Q346 Neurology
A 19-year-old girl about to take an exam, falls to
the floor with forcefully closed eyes and her
body shakes rapidly. The episode lasts a minute
until she tells someone to take her to the ED. She
describes the event to the physician. What is the
diagnosis?

A) Caffeine intoxication
B) Tonic-clonic seizure
C) Psychogenic nonepileptic seizure
Q346 Neurology
A 19-year-old girl about to take an exam, falls to
the floor with forcefully closed eyes and her
body shakes rapidly. The episode lasts a minute
until she tells someone to take her to the ED. She
describes the event to the physician. What is the
diagnosis?

A) Caffeine intoxication
B) Tonic-clonic seizure
C) Psychogenic nonepileptic seizure
Q346 Neurology
A 19-year-old girl about to take an exam, falls to
the floor with forcefully closed eyes and her
body shakes rapidly. The episode lasts a minute
until she tells someone to take her to the ED. She
signs that it's not a "real" seizure, but psychogenic (a type of
conversion disorder") include - forceful eye closure, rapid
describes the event to the physician. What
reorienting, and memory recall of the event. Video EEG shows is the
diagnosis?
lack of epileptic activity. Psychiatric assessment is important -
patients often have comorbid psychiatric conditions.
A) Caffeine intoxication
B) Tonic-clonic seizure
C) Psychogenic nonepileptic seizure
Q347 Neurology
A 5-year-old boy undergoes a seizure and now
cannot move his right arm or leg. CT and MRI are
normal. What is the most likely outcome?

A) Self-resolution within 36 hours


B) Permanent damage
Q347 Neurology
A 5-year-old boy undergoes a seizure and now
cannot move his right arm or leg. CT and MRI are
normal. What is the most likely outcome?

A) Self-resolution within 36 hours


B) Permanent damage
Q347 Neurology
A 5-year-old boy undergoes a seizure and now
cannot move his right arm or leg. CT and MRI are
normal. What is the most likely outcome?

A) Self-resolution within 36 hours


B) Permanent damage
Todd's paralysis - a neurological condition
experienced by individuals with epilepsy, in which a
seizure is followed by a brief period of temporary
paralysis. Resolves from 30 minutes to 36 hours.
Q348 Neurology
Generalized convulsive status epilepticus is
treated with IV benzodiazepines, followed by:

A) Carbamazepine
B) Lumbar puncture
C) Levetiracetam
Q348 Neurology
Generalized convulsive status epilepticus is
treated with IV benzodiazepines, followed by:

A) Carbamazepine
B) Lumbar puncture
C) Levetiracetam
Q348 Neurology
Generalized convulsive status epilepticus is
treated with IV benzodiazepines, followed by:

A) Carbamazepine
B) Lumbar puncture
C) Levetiracetam After giving the benzo, a nonbenzodiazepine
antiepileptic medication should be given to
prevent seizure recurrence (e.g., levetiracetam,
fosphenytoin, or valproic acid). NOT
carbamazepine - has a slow onset of action, not to
prevent GCSE recurrence.
Q349 Neurology
A 2-year-old boy is brought to the ED due to
seizures. He is now alert and playful, but
temperature is 102.5F. He has no cough, vomiting,
or diarrhea, and the rest of the PE is normal.
After acetaminophen what is the next step?

A) Hospitalization for observation


B) EEG
C) Discharge home
Q349 Neurology
A 2-year-old boy is brought to the ED due to
seizures. He is now alert and playful, but
temperature is 102.5F. He has no cough, vomiting,
or diarrhea, and the rest of the PE is normal.
After acetaminophen what is the next step?

A) Hospitalization for observation


B) EEG
C) Discharge home
Q349 Neurology
A 2-year-old boy is brought to the ED due to
seizures. He is now alert and playful, but
temperature is 102.5F. He has no cough, vomiting,
or diarrhea, and the rest of the PE is normal.
After acetaminophen what is the next step?

A) Hospitalization for observation


B) EEG Febrile seizures are common and generally benign; occur with
fevers in kids ages 6 months to 5 years. Patients may have a
C) Discharge home postictal period (e.g., drowsiness), return to baseline within
minutes. Neuro exam is normal; diagnosis is clinical.
Q350 Neurology
A woman with DM has back and leg pain
associated with weakness, paresthesia in
the feet, an upgoing plantar reflex, and a
right sided hyperactive knee jerk. These
symptoms have been developing over the
last 2 months. What is the next step?

A) Nerve conduction studies


B) MRI of the spine
Q350 Neurology
A woman with DM has back and leg pain
associated with weakness, paresthesia in
the feet, an upgoing plantar reflex, and a
right sided hyperactive knee jerk. These
symptoms have been developing over the
last 2 months. What is the next step?

A) Nerve conduction studies


B) MRI of the spine
Q350 Neurology
A woman with DM has back and leg pain
associated with weakness, paresthesia in
the feet, an upgoing plantar reflex, and a
right sided hyperactive knee jerk. These
symptoms have been developing over the
last 2 months. This is not mere diabetic neuropathy - first of all,
that does not develop so rapidly (i.e., over
months); secondly, diabetic neuropathy usually
causes a symmetric "stocking and glove" pattern of
A) Nerve conduction studies sensory loss and proprioception. Motor weakness
B) MRI of the spine doesn't usually occur and when it does, it occurs
after years of severe disease. need to assess for a
spinal lesion (e.g., malignancy) causing symptoms.
Q351 Neurology
Low back pain radiating to the
butt, lateral thigh, lateral shin
& dorsal foot, along with
weakness in foot dorsiflexion
is due to a(n):

A) L2-L4 lesion
B) L5 lesion
C) S1 lesion
D) Common fibular lesion
Q351 Neurology
Low back pain radiating to the
butt, lateral thigh, lateral shin
& dorsal foot, along with
weakness in foot dorsiflexion
is due to a(n):

A) L2-L4 lesion
B) L5 lesion
C) S1 lesion
D) Common fibular lesion
Q351 Neurology
Low back pain radiating to the
butt, lateral thigh, lateral shin
& dorsal foot, along with
weakness in foot dorsiflexion
is due to a(n):

A) L2-L4 lesion
B) L5 lesion
C) S1 lesion Wouldn’t cause low back pain or butt pain!!
D) Common fibular lesion
Q352 Neurology
A 70-year-old man with a history of lung cancer
presents with 2 weeks of progressive leg weakness
and increased urinary hesitancy. He has had back pain
for months, but only now is it radiating to his legs.
Scattered sensory loss is noted in the lower
extremities and perianal skin. What is the diagnosis?

A) Conus medullaris syndrome


B) Cauda equina syndrome
Q352 Neurology
A 70-year-old man with a history of lung cancer
presents with 2 weeks of progressive leg weakness
and increased urinary hesitancy. He has had back pain
for months, but only now is it radiating to his legs.
Scattered sensory loss is noted in the lower
extremities and perianal skin. What is the diagnosis?

A) Conus medullaris syndrome


B) Cauda equina syndrome
Q352 Neurology
A 70-year-old man with a history of lung cancer
presents with 2 weeks of progressive leg weakness
and increased urinary hesitancy. He has had back pain
for months, but only now is it radiating to his legs.
Scattered sensory loss is noted in the lower
extremities and perianal skin. What is the diagnosis?
More symmetrical weakness, usually
A) Conus medullaris syndrome due to sudden onset (e.g., trauma)

B) Cauda equina syndromedeficit in legs, saddle anesthesia, loss of rectal


Cauda equina syndrome - usually asymmetric motor

sphincter and bladder; diagnose with MRI; tx:


emergency surgery
Q353 Neurology
A woman with RA develops weakness,
paresthesias, and hyperreflexia after
intubation. This is due to:

A) Critical illness polyneuropathy


B) Instability of the atlantoaxial joint
Q353 Neurology
A woman with RA develops weakness,
paresthesias, and hyperreflexia after
intubation. This is due to:

A) Critical illness polyneuropathy


B) Instability of the atlantoaxial joint
Q353 Neurology
A woman with RA develops weakness,
paresthesias, and hyperreflexia after
intubation. This is due to:

A) Critical illness polyneuropathy


B) Instability of the atlantoaxial joint
Patient with RA may have subluxation (or misalignment) of the atlantoaxial
joints; forced neck extension (e.g., during intubation) can worsen the
subluxation, leading to cord compression and progressive cervical myelopathy.
Upper motor signs (e.g., hyperreflexia) are present and sensory changes.
(complete dislocation can result in paralysis and death). Diagnosis: MRI of the
cervical spine showing separation of S1 and S2. Tx: surgery.
Q354 Neurology
A 66-year-old man presents with progressive
neck pain, gait disturbances, LMN signs in the
arms (i.e., weakness and diminished reflexes)
with UMN in the legs and feet (i.e., increased
reflexes and spasticity). What is the diagnosis?

A) Demyelinating polyneuropathy
B) Cervical spondylotic myelopathy
Q354 Neurology
A 66-year-old man presents with progressive
neck pain, gait disturbances, LMN signs in the
arms (i.e., weakness and diminished reflexes)
with UMN in the legs and feet (i.e., increased
reflexes and spasticity). What is the diagnosis?

A) Demyelinating polyneuropathy
B) Cervical spondylotic myelopathy
Q354 Neurology
Spondylosis is a degenerative
A 66-year-old man presents with progressive
condition associated w/
neck pain, gait disturbances, LMN signs in the
spinal canal narrowing; some
patients can even have direct
arms (i.e., weakness and diminishedcompression
reflexes) of the spinal
with UMN in the legs and feet (i.e., increased
cord resulting in cervical
spondylotic myelopathy.
reflexes and spasticity). What is the diagnosis?
(Cervical spinal root lesion
leads to LMN signs in the
arms; descending tract
A) Demyelinating polyneuropathy compression leads to UMN
B) Cervical spondylotic myelopathyincreased tone - in the lower
signs - hyperreflexia,

extremity. (Lhermitte sign


may also be seen)
Q355 Neurology
A 35-year-old woman was playing volleyball when
she experienced neck and upper back pain. She now
has constant dull pain and intermittent sharp pain
with neck movement that radiates to the right
hand. Vital signs are normal; there are no other
sensory/motor deficits. What is the next step?

A) MRI of the cervical spine


B) Avoidance of triggering activities and NSAIDs
Q355 Neurology
A 35-year-old woman was playing volleyball when
she experienced neck and upper back pain. She now
has constant dull pain and intermittent sharp pain
with neck movement that radiates to the right
hand. Vital signs are normal; there are no other
sensory/motor deficits. What is the next step?

A) MRI of the cervical spine


B) Avoidance of triggering activities and NSAIDs
Q355 Neurology
A 35-year-old woman was playing volleyball when
she experienced neck and upper back pain. She now
has constant dull pain and intermittent sharp pain
with neck movement that radiates to the right
hand. Vital signs are normal; there are no other
sensory/motor deficits. What is the next step?
Diagnosis of cervical radiculopathy is clinical,

A) MRI of the cervical spine


and most patients improve with avoidance of
triggering activities and NSAIDs.

B) Avoidance of triggering activities and NSAIDs


Q356 Neurology
A 37-year-old woman has bilateral LE
weakness associated with UMN signs (e.g.,
increased deep tendon reflexes), urinary
incontinence, and sensation deficits below
the level of the umbilicus. What is the cause?

A) Compression of the cauda equina


B) Segmental inflammation of the spinal cord
Q356 Neurology
A 37-year-old woman has bilateral LE
weakness associated with UMN signs (e.g.,
increased deep tendon reflexes), urinary
incontinence, and sensation deficits below
the level of the umbilicus. What is the cause?

A) Compression of the cauda equina


B) Segmental inflammation of the spinal cord
Q356 Neurology
A 37-year-old woman has bilateral LE
weakness associated with UMN signs (e.g.,
increased deep tendon reflexes), urinary
incontinence, and sensation deficits below
the level of the umbilicus. What is the cause?
(not cauda equina, because sensory loss is below the umbilicus - T10!!!)

B) Segmental inflammation of the spinal cord


Transverse myelitis (immune infiltration of inflammatory cells into the spinal cord, leading to
neuron death and demyelination); most cases follow a recent infection (e.g., GI, URI) although it is
associated with MS and sarcoidosis. Inflammation usually affects the thoracic region. Tx: IV
steroids, & often plasmapheresis.
Q357 Neurology
A 66-year-old man was in a car accident and
how has numbness, tingling, and marked
weakness in both upper extremities. He can
move his legs. What is the diagnosis?

A) Anterior cord syndrome


B) Posterior cord syndrome
C) Central cord syndrome
Q357 Neurology
A 66-year-old man was in a car accident and
how has numbness, tingling, and marked
weakness in both upper extremities. He can
move his legs. What is the diagnosis?

A) Anterior cord syndrome


B) Posterior cord syndrome
C) Central cord syndrome
Q357 Neurology
A 66-year-old man was in a car accident and
how has numbness, tingling, and marked
weakness in both upper extremities. He can
move his legs. What is the diagnosis?

A) Anterior cord syndrome bilateral weakness that is proportionate


in the upper and lower extremities

B) Posterior cord syndrome Loss in vibration/proprioception

C) Central cord syndrome


Hyperextension injuries can lead to central cord syndrome; small lesions can affect only the lateral spinothalamic tracts - causing
sensory deficits in the upper extremities (pain/temperature/numbness) [similar presentation as Syringomyelia!!]; larger lesions can
affect the lower extremities, but not as greatly because the upper extremity motor fibers are located closer to the central part of the
spinal cord.
Q358 Neurology
A 50-year-old man in a car accident states that
he has neck pain; BP = 84/52. There are
abrasions and bruises over his chest but no other
obvious signs of bleeding. Heart sounds are
normal, pulse = 50/min. He cannot move any
extremities. What is the cause of the low BP?

A) Intraperitoneal hemorrhage
B) Injury to descending tracts
Q358 Neurology
A 50-year-old man in a car accident states that
he has neck pain; BP = 84/52. There are
abrasions and bruises over his chest but no other
obvious signs of bleeding. Heart sounds are
normal, pulse = 50/min. He cannot move any
extremities. What is the cause of the low BP?

A) Intraperitoneal hemorrhage
B) Injury to descending tracts
Q358 Neurology
A 50-year-old man in a car accident states that
[hemorrhage would have led to
he has neck pain; BP = 84/52. There are
tachycardia]
abrasions and bruises over his chest but no other
Acute spinal cord injury manifests
with loss of motor & sensation;
obvious signs of bleeding. Heartlesions
sounds are
above T1 can cause
normal, pulse = 50/min. He cannot move any
neurogenic shock due to
interruption of the descending
extremities. What is the cause of the low
sympathetic fibersBP?
(results in
unopposed parasympathetic
stimulation - the intact vagus -
A) Intraperitoneal hemorrhagehypotension, and bradycardia).
B) Injury to descending tracts of peripheral vasoconstriction).
(Hypothermia may result due to lack
Q359 Neurology
A 60-year-old woman has intermittent right-
sided neck and shoulder pain worse with neck
movement and associated with forearm
numbness. Lower extremity is normal bilaterally.
What will most likely be seen on Xray?

A) Cervical spondylosis
B) Atlantoaxial subluxation
Q359 Neurology
A 60-year-old woman has intermittent right-
sided neck and shoulder pain worse with neck
movement and associated with forearm
numbness. Lower extremity is normal bilaterally.
What will most likely be seen on Xray?

A) Cervical spondylosis
B) Atlantoaxial subluxation
Q359 Neurology
A 60-year-old woman has intermittent right-
sided neck and shoulder pain worse with neck
movement and associated with forearm
numbness. Lower extremity is normal bilaterally.
What will most likely be seen on Xray?
Presentation suspicious for cervical radiculopathy (usually in
older adults due to acute cervical disc herniation or

A) Cervical spondylosis underlying cervical spondylosis [a degenerative condition of


the spine marked by osteophyte formation in the facet

B) Atlantoaxial subluxation
joints] - leads to foramen narrowing).
Evaluation of suspected cervical radiculopathy
[Atlantoaxial subluxation - in RA patients; patients get spastic paresis, usually begins with MRI of the cervical spine to
Lhermitte sign, and lower extremity manifestations; episodic visualize anatomy
neck/shoulder pain and forearm paresthesia would be very unusual]
Q360 Neurology
A 19-year-old man is in a motorcycle accident. CT
scan of the cervical spine shows a mild displaced
fracture of the right C5 lamina. The patient has
mild weakness of right wrist extension and loss of
pinprick sensation of the right thumb. What is the
next step?

A) Nerve conduction study of the right arm


B) CT scan of thoracic and lumbar spine
Q360 Neurology
A 19-year-old man is in a motorcycle accident. CT
scan of the cervical spine shows a mild displaced
fracture of the right C5 lamina. The patient has
mild weakness of right wrist extension and loss of
pinprick sensation of the right thumb. What is the
next step?

A) Nerve conduction study of the right arm


B) CT scan of thoracic and lumbar spine
Q360 Neurology
A 19-year-old man is in a motorcycle accident. CT
scan of the cervical spine shows a mild displaced
fracture of the right C5 lamina. The patient has
mild weakness of right wrist extension and loss of
pinprick sensation of the right thumb. What is the
next step? IfnotyouXray)!
see a single vertebral fracture, you need to image the entire spine (with CT scan,
(Nerve conduction studies are to localize a nerve injury/compression to
guide treatment, e.g., carpal tunnel syndrome with carpal tunnel release)

A) Nerve conduction study of the right arm


B) CT scan of thoracic and lumbar spine
Q361 Neurology
A 25-year-old woman has weakness and numbness
in both arms and legs that has progressively
worsened over the last few days. She has no fever,
there was no trauma, and she has no medical
condition. MRI shows T2 signal in the cervical spinal
cord. What should be given?

A) Immunoglobulin
B) Steroids
Q361 Neurology
A 25-year-old woman has weakness and numbness
in both arms and legs that has progressively
worsened over the last few days. She has no fever,
there was no trauma, and she has no medical
condition. MRI shows T2 signal in the cervical spinal
cord. What should be given?

A) Immunoglobulin
B) Steroids
Q361 Neurology
A 25-year-old woman has weakness and numbness
in both arms and legs that has progressively
worsened over the last few days. She has no fever,
there was no trauma, and she has no medical
condition. MRI shows T2 signal in the cervical spinal
cord. What should be given?

A) Immunoglobulin
B) Steroids hyperintensity suggests transverse myelitis (immune mediated - so give steroids, IV;
rapidly progressive bilateral motor and sensory deficits of the extremities, T2

plasmapheresis if no response)
Q362 Neurology
The best test to screen for cervical spine injury
(e.g., if spinal tenderness, neurologic deficit) is:

A) X-ray
B) CT without contrast
C) MRI
Q362 Neurology
The best test to screen for cervical spine injury
(e.g., if spinal tenderness, neurologic deficit) is:

A) X-ray
B) CT without contrast
C) MRI
Q362 Neurology
The best test to screen for cervical spine injury
(e.g., if spinal tenderness, neurologic deficit) is:

A) X-ray
B) CT without contrast
C) MRI CT is more sensitive than Xray (98% vs. 52%); MRI is
only done when CT scan suggests ligamentous or
spinal cord injury to better visualize the soft tissues
- CT for Cancer, infection, bones, blood
Q363 Neurology
A 67-year-old woman is brought to the ED due to
bilateral leg weakness/tingling for 5 hours. Spinal
tenderness is present at T6. Upper extremity
strength is intact. MRI reveals fluid collection
spanning T6-T8. What is the next step?

A) Lumbar puncture
B) Emergent laminectomy and decompression
Q363 Neurology
A 67-year-old woman is brought to the ED due to
bilateral leg weakness/tingling for 5 hours. Spinal
tenderness is present at T6. Upper extremity
strength is intact. MRI reveals fluid collection
spanning T6-T8. What is the next step?

A) Lumbar puncture
B) Emergent laminectomy and decompression
Q363 Neurology
A 67-year-old woman is brought to the ED due to
bilateral leg weakness/tingling for 5 hours. Spinal
tenderness is present at T6. Upper extremity
strength is intact. MRI reveals fluid collection
spanning T6-T8. What is the next step?

A) Lumbar puncture LP not performed because it doesn't provide any info;


also, risk of introducing infection into the CSF.

B) Emergent laminectomy and decompression


Symptoms are concerning for spinal epidural abscess - a neurologic emergency requiring
laminectomy.
Q364 Neurology
A 30-year-old woman has loss of pain and
temperature, along with diminished strength on
the upper limbs. Vibration/proprioception is
preserved, and her legs have no abnormalities.
What is the cause?

A) Fluid filled cavity in the central spinal cord


B) Hemorrhage into the spinal cord
Q364 Neurology
A 30-year-old woman has loss of pain and
temperature, along with diminished strength on
the upper limbs. Vibration/proprioception is
preserved, and her legs have no abnormalities.
What is the cause?

A) Fluid filled cavity in the central spinal cord


B) Hemorrhage into the spinal cord
Q364 Neurology
A 30-year-old woman has loss of pain and
temperature, along with diminished strength on
the upper limbs. Vibration/proprioception is
preserved, and her legs have no abnormalities.
What is the cause?
Syringomyelia - most associated with Chiari type 1 malformation, but also seen in spinal cord
infection, neoplasms, or trauma. Has a cape like distribution of pain/temperature loss (due to
crossing spinothalamic tracts) dorsal columns unaffected. Syrinx enlargement can affect the lateral
corticospinal tracts (of the arms, more central) - weakness more in the upper extremities.

A) Fluid filled cavity in the central spinal cord


B) Hemorrhage into the spinal cord
Q365 Neurology
What is the first step in a patient with cervical
spine trauma with potentially diaphragmatic
paralysis and imminent respiratory failure?

A) Intubation
B) Cricothyroidotomy
Q365 Neurology
What is the first step in a patient with cervical
spine trauma with potentially diaphragmatic
paralysis and imminent respiratory failure?

A) Intubation
B) Cricothyroidotomy
Q365 Neurology
What is the first step in a patient with cervical
spine trauma with potentially diaphragmatic
paralysis and imminent respiratory failure?
ABC's come first with all trauma!
A) Intubation First go with intubation (i.e.,
B) Cricothyroidotomy airway)- even in unstable
cervical spine injury!! Only go
with cricothyroidotomy if
intubation fails (e.g., severe
maxillofacial trauma).
Q366 Neurology
A harmful stimulus (e.g., retained urinary catheter,
constipation) below the level of the lesion in a
patient with spinal cord injury may lead to severe
hypertension, fecal impaction, and other symptoms
due to:

A) an intact parasympathetic response


B) an unregulated sympathetic response
C) both A and B
Q366 Neurology
A harmful stimulus (e.g., retained urinary catheter,
constipation) below the level of the lesion in a
patient with spinal cord injury may lead to severe
hypertension, fecal impaction, and other symptoms
due to:

A) an intact parasympathetic response


B) an unregulated sympathetic response
C) both A and B
Q366 Neurology
A harmful stimulus (e.g., retained urinary catheter,
constipation) below the level of the lesion in a patient
with spinal cord injury may lead to severe hypertension,
fecal impaction, and other symptoms due to:
Autonomic dysreflexia - seen in spinal cord injury patients; the parasympathetic
A) an intact parasympathetic response
response is intact - which can cause bradycardia and vasodilation (e.g., facial flushing);
the disrupted modulation leads to unregulated sympathetic response - above T6, this can
B) anboth
involve unregulated sympathetic
splanchnic and peripheral response
vasculatures, causing severe hypertension that
overwhelms any parasympathetic compensation (prolonged hypertension can cause
headache and hemorrhage). Tx: place patient in upright position to encourage
orthostatic BP reduction, remove noxious stimulus (e.g., urinary catheter, tight clothes),
and treat the hypertension (e.g., nitrates).
Q367 Neurology
The crossed straight leg raise test has a high
specificity for:

A) Disc herniation
B) Spondylolisthesis
Q367 Neurology
The crossed straight leg raise test has a high
specificity for:

A) Disc herniation
B) Spondylolisthesis
Q367 Neurology
The crossed straight leg raise test has a high
specificity for:
The crossed straight leg raise test (exacerbation of pain
A) Disc herniation with passive lifting of the unaffected leg) has poor
sensitivity but high specificity for disc herniation.
B) Spondylolisthesis
Q367 Neurology
The crossed straight leg raise test has a high
specificity for:
The crossed straight leg raise test (exacerbation of pain
A) Disc herniation with passive lifting of the unaffected leg) has poor
sensitivity but high specificity for disc herniation.
B) Spondylolisthesis

Don’t confuse with


Kernig sign for
meningitis (pain on
passive extension of
knee on flexed hip)
Q368 Neurology
A 79-year-old woman has weakness in upper and
lower extremities, intrinsic hand muscle atrophy,
stiffness/hyperreflexia in her legs, and Lhermitte
sign positive; what is the diagnosis?

A) Multiple sclerosis
B) Compressed spinal cord
Q368 Neurology
A 79-year-old woman has weakness in upper and
lower extremities, intrinsic hand muscle atrophy,
stiffness/hyperreflexia in her legs, and Lhermitte
sign positive; what is the diagnosis?

A) Multiple sclerosis
B) Compressed spinal cord
Q368 Neurology
A 79-year-old woman has weakness in upper and
lower extremities, intrinsic hand muscle atrophy,
stiffness/hyperreflexia in her legs, and Lhermitte
sign positive; what is the diagnosis?
LMN signs: FAR = fasciculations, atrophy, reduced tone

A) Multiple sclerosis
B) Compressed spinal cord
Cervical myelopathy has an insidious onset; compression of spinal cord (descending tracts) leads to
UMN signs (e.g, hyperreflexia) below the lesion; compression of roots leads to LMN findings (e.g.,
atrophy, hyporeflexia) in above the lesion. Lhermitte sign can occur when neck flexion compresses
and activates the ascending spinothalamic pain tracts - but also seen in MS (not seen in older
ladies; also, LMN signs such as atrophy are absent).
Q369 Neurology
A 65-year-old man with pain in the low back and
legs has improvement of symptoms with walking
downhill or flexion of the back. What will reveal the
most likely diagnosis?

A) Bone scan
B) MRI
C) HLA-B27
Q369 Neurology
A 65-year-old man with pain in the low back and
legs has improvement of symptoms with walking
downhill or flexion of the back. What will reveal the
most likely diagnosis?

A) Bone scan
B) MRI
C) HLA-B27
Q369 Neurology
A 65-year-old man with pain in the low back and
legs has improvement of symptoms with walking
downhill or flexion of the back. What will reveal the
most likely diagnosis? (Flexion of the spine
causes widening of the
spinal canal; extension
causes narrowing of the
A) Bone scan canal.)

B) MRI bony anatomy, neural structures and soft tissue


MRI can confirm diagnosis of lumbar stenosis as it demonstrates

C) HLA-B27 ankylosing spondylitis


Q370 Neurology
A man who underwent thoracic aortic aneurysm
repair now has lower extremity, bilateral flaccid
paralysis, loss of pain/temperature/crude touch,
and urinary retention. Proprioception is intact.
What happened?

A) Physical compression
B) Ischemia
Q370 Neurology
A man who underwent thoracic aortic aneurysm
repair now has lower extremity, bilateral flaccid
paralysis, loss of pain/temperature/crude touch,
and urinary retention. Proprioception is intact.
What happened?

A) Physical compression
B) Ischemia
Q370 Neurology
A man who underwent thoracic aortic aneurysm
repair now has lower extremity, bilateral flaccid
paralysis, loss of pain/temperature/crude touch,
and urinary retention. Proprioception is intact.
What happened?

A) Physical compression
B) Ischemia especially of the anterior cord – leading to anterior cord syndrome
Thoracic aortic aneurysm repair can cause spinal cord ischemia,

(distal, bilateral flaccid paralysis; loss of pain/temperature and crude


touch sensation; and urinary retention).
Q371 Neurology
A 34-year-old woman with a history of a neck pain
after she fell a month ago has left shoulder pain. PE
shows decreased pinprick sensation of the left
thumb and index finger; there is also weakness with
left elbow flexion and decreased biceps reflex.
What is the cause?

A) Median nerve compression


B) Spinal nerve root compression
Q371 Neurology
A 34-year-old woman with a history of a neck pain
after she fell a month ago has left shoulder pain. PE
shows decreased pinprick sensation of the left
thumb and index finger; there is also weakness with
left elbow flexion and decreased biceps reflex.
What is the cause?

A) Median nerve compression


B) Spinal nerve root compression
Q371 Neurology
A 34-year-old woman with a history of a neck pain
after she fell a month ago has left shoulder pain. PE
shows decreased pinprick sensation of the left
thumb and index finger; there is also weakness with
left elbow flexion and decreased biceps reflex.
What is the cause?

A) Median nerve compression Findings are not consistent with


a single peripheral nerve lesion
B) Spinal nerve root compression but a distribution; in this case, a
C6 nerve root lesion
Q372 Neurology
A patient with acute spinal cord injury is stabilized.
Neurosurgery is on their way. What is the next
step?

A) Oral prednisone
B) Bladder cath
Q372 Neurology
A patient with acute spinal cord injury is stabilized.
Neurosurgery is on their way. What is the next
step?

A) Oral prednisone
B) Bladder cath
Q372 Neurology
A patient with acute spinal cord injury is stabilized.
Neurosurgery is on their way. What is the next
step?

A) Oral prednisone
B) Bladder cath
patients with traumatic spinal cord injury should have a bladder cath placed
to prevent distention and possible injury. (certain patients may be given
methylprednisone - but not oral! remain NPO until neurosurgical consultation
determines whether immediate surgery is indicated)
Q373 Neurology
Contralateral leg weakness without pupillary
involvement is seen with:

A) Uncal herniation
B) Subfalcine herniation
Q373 Neurology
Contralateral leg weakness without pupillary
involvement is seen with:

A) Uncal herniation
B) Subfalcine herniation
Q373 Neurology
Contralateral leg weakness without pupillary
involvement is seen with:

A) Uncal herniation
B) Subfalcine herniation
-Subfalcine herniation - cingulate gyrus is displaced under
the falx cerebri; does not cause pupillary involvement but
ACA compression can lead to contralateral leg weakness
-Uncal herniation - ipsilateral fixed and dilated pupil (due to
compression of ipsilateral oculomotor nerve)
-(tonsillar - fixed misposition pupils as both sympathetic and
parasympathetic innervation is disrupted)
Q374 Neurology
Prolonged concussion symptoms (e.g., >4 weeks of
headache and sleep disturbances) with otherwise
intact neuro exam should be managed with:

A) lumbar puncture
B) MRI
C) symptomatic care only and activity as tolerated
Q374 Neurology
Prolonged concussion symptoms (e.g., >4 weeks of
headache and sleep disturbances) with otherwise
intact neuro exam should be managed with:

A) lumbar puncture
B) MRI
C) symptomatic care only and activity as tolerated
Q374 Neurology
Prolonged concussion symptoms (e.g., >4 weeks of
headache and sleep disturbances) with otherwise
intact neuro exam should be managed with:
Concussion pathophysiology: transient disturbance of

A) lumbar puncture
normal neuronal function.

B) MRI
C) symptomatic care only and activity as tolerated
Patients with post-concussion syndrome generally improve within
3 months
Q375 Neurology
A man in a motorcycle accident does not answer
questions or open his eyes but withdraws to pain.
Brain imaging shows cerebral edema but no
hematomas or fractures. How should intracranial
reduction be performed?

A) Steroids
B) Lumbar puncture
C) Hypertonic saline
Q375 Neurology
A man in a motorcycle accident does not answer
questions or open his eyes but withdraws to pain.
Brain imaging shows cerebral edema but no
hematomas or fractures. How should intracranial
reduction be performed?

A) Steroids
B) Lumbar puncture
C) Hypertonic saline
Q375 Neurology
A man in a motorcycle accident does not answer
questions or open his eyes but withdraws to pain.
Brain imaging shows cerebral edema but no
hematomas or fractures. How should intracranial
reduction be performed?
How does hyperventilation help lower ICP? By Osmotic therapy (hypertonic saline or mannitol) is part
causing cerebral washout of CO2, leading to of the initial therapy for elevated ICP - this draws water
A) Steroids
vasoconstriction and decreased cerebral blood
flow!
out of the edematous brain tissue, thereby reducing
parenchymal volume and overall ICP. also head
B) Lumbar puncture elevation (increase venous flow) and sedation (reduces

C) Hypertonic saline
metabolic demand).; (steroids increase mortality in TBI;
LB isn't done because it can lead to brain herniation -
drainage is via an external ventricular shunt).
Q376 Neurology
Patients with concussion (e.g., transient
disorientation, headache, dizziness) can return to
full activity in:

A) 24 hours
B) 1 week
Q376 Neurology
Patients with concussion (e.g., transient
disorientation, headache, dizziness) can return to
full activity in:

A) 24 hours
B) 1 week
Q376 Neurology
Patients with concussion (e.g., transient
disorientation, headache, dizziness) can return to
full activity in:

A) 24 hours
B) 1 week
after concussion, rest is indicated for at least 24
hours, gradually increasing activity level as long as
they remain asymptomatic, potentially returning
fully (even to contact sports) in 1 week
Q377 Neurology
A 5-year-old boy hit his head on the floor with mild
pain, got up and walked normally for several
minutes, and then developed headache/vomiting
and then somnolence. He then becomes braindead
several hours later. What happened?

A) Subdural hematoma
B) Epidural hematoma
Q377 Neurology
A 5-year-old boy hit head on the floor with mild
pain, got up and walked normally for several
minutes, and then developed headache/vomiting
and then somnolence. He then becomes braindead
several hours later. What happened?

A) Subdural hematoma
B) Epidural hematoma
Q377 Neurology
A 5-year-old boy hit head on the floor with mild
pain, got up and walked normally for several
minutes, and then developed headache/vomiting
and then somnolence. He then becomes braindead
several hours later. What happened?
(also shaken baby syndrome)

A) Subdural hematoma – elderly & alcoholics


B) Epidural hematoma – lucid period (minutes to
hours)
Q378 Neurology
Which of the following is most associated with
unconsciousness and decorticate posturing?

A) Acute dystonia
B) Scorpion bites
C) Strychnine
D) Tetanus
E) Central herniation
Q378 Neurology
Which of the following is most associated with
unconsciousness and decorticate posturing?

A) Acute dystonia
B) Scorpion bites
C) Strychnine
D) Tetanus
E) Central herniation
Q378 Neurology
Which of the following is most associated with
unconsciousness and decorticate posturing?

A) Acute dystonia - antipsychotics; involuntary contractions/spasms; patient conscious


B) Scorpion bites - fasciculations and jerking of extremities; patient conscious
C) Strychnine - found in illicit drugs; episodic contractions in fully awake patient
D) Tetanus - painful muscle contractions; patient awake
E) Central herniation - diencephalon & midbrain displaced caudally due to
hemorrhage; causes damage brainstem (Cushing triad, unconsciousness, abnormal posturing)
Q379 Neurology
What can be observed even in a brain-dead patient?

A) Pupillary light reflect


B) HR increase after atropine
C) Deep tendon reflexes
Q379 Neurology
What can be observed even in a brain-dead patient?

A) Pupillary light reflect


B) HR increase after atropine
C) Deep tendon reflexes
Q379 Neurology
What can be observed even in a brain-dead patient?

A) Pupillary light reflect


B) HR increase after atropine
C) Deep tendon reflexes – originate from the spinal
cord
Q380 Neurology
After a hard hiking fall, a 26-year-old female has
neck pain and unilateral headache. PE shows ptosis
and miosis of the left eye. Neuro exam shows
normal motor strength, deep tendon reflexes, and
sensation in all extremities. What happened?

A) Migraine
B) Carotid artery dissection
Q380 Neurology
After a hard hiking fall, a 26-year-old female has
neck pain and unilateral headache. PE shows ptosis
and miosis of the left eye. Neuro exam shows
normal motor strength, deep tendon reflexes, and
sensation in all extremities. What happened?

A) Migraine
B) Carotid artery dissection
Q380 Neurology
After a hard hiking fall, a 26-year-old female has
neck pain and unilateral headache. PE shows ptosis
and miosis of the left eye. Neuro exam shows
normal motor strength, deep tendon reflexes, and
sensation in all extremities. What happened?
carotid artery dissection – cause of stroke in young
patients; can occur after infections or minor trauma (risk

A) Migraine factors: Ehlers-Danlos, smokers, uncontrolled hypertension);


dissection leads to an intramural hematoma, leading to

B) Carotid artery dissection


obstruction/thrombus -> ischemia; present with unilateral
head and neck pain; sometimes transient vision loss;
ipsilateral partial Horner syndrome - ptosis, myosis, no
anhidrosis; tx: tpa if within 4.5 hours, antiplatelet (e.g.,
aspirin) +/- anticoagulation
Q381 Neurology
Which head injury would be an indication for CT
scan without contrast (vs. sending home):

A) Somnolence
B) Loss of consciousness
C) Fall from 5ft/3ft high (adults/babies)
D) Vomiting
E) Severe headache
F) All of the above
Q381 Neurology
Which head injury would be an indication for CT
scan without contrast (vs. sending home):

A) Somnolence
B) Loss of consciousness
C) Fall from 5ft/3ft high (adults/babies)
D) Vomiting
E) Severe headache
F) All of the above
Q381 Neurology
Which head injury would be an indication for CT
scan without contrast (vs. sending home):

A) Somnolence
B) Loss of consciousness
C) Fall from 5ft/3ft high (adults/babies)
D) Vomiting Observation for 4-6 hours may be considered for a

E) Severe headache mental status changes or sign of skull fracture; but


patient with mild/improving condition and no

F) All of the above get CT scan if symptoms worsen during this


observation period
Q382 Neurology
Rapidly progressive, bilateral lower extremity
weakness with UMN signs (e.g., hyperreflexia),
sensory defects, and bladder dysfunction are seen
in:

A) immune destruction of the spinal cord


B) acute inflammatory demyelination
Q382 Neurology
Rapidly progressive, bilateral lower extremity
weakness with UMN signs (e.g., hyperreflexia),
sensory defects, and bladder dysfunction are seen
in:

A) immune destruction of the spinal cord


B) acute inflammatory demyelination
Q382 Neurology
Rapidly progressive, bilateral lower extremity
weakness with UMN signs (e.g., hyperreflexia),
sensory defects, and bladder dysfunction are seen
in:
Transverse myelitis – immune destruction of the spinal cord leading to
UMN signs, motor/sensory deficits, and bladder dysfunction
A) immune destruction of the spinal cord
B) acute inflammatory demyelination
No UMN signs in GBS!!
Q383 Neurology
A 58-year-old man is brought to the ED after he was
found unresponsive. He has severe neurologic
impairment but no major CT scan findings or
obvious signs of trauma. What happened?

A) Punctate hemorrhages in the white matter


B) Bleeding in the basal cisterns
Q383 Neurology
A 58-year-old man is brought to the ED after he was
found unresponsive. He has severe neurologic
impairment but no major CT scan findings or
obvious signs of trauma. What happened?

A) Punctate hemorrhages in the white matter


B) Bleeding in the basal cisterns
Q383 Neurology
A 58-year-old man is brought to the ED after he was
found unresponsive. He has severe neurologic
impairment but no major CT scan findings or
obvious signs of trauma. What happened?

A) Punctate hemorrhages in the white matter


Diffuse axonal injury causes severe
B) Bleeding in the basal cisterns neurologic impairment without major
Subarachnoid hemorrhage CT findings; MRI is sensitive - shows
minute punctate hemorrhages in the
white matter and blurring of the gray-
white interface
Q384 Neurology
A boy with poor linear growth, puberty delay,
worsening headaches, papilledema most likely has:

A) Medulloblastoma
B) Craniopharyngioma
Q384 Neurology
A boy with poor linear growth, puberty delay,
worsening headaches, papilledema most likely has:

A) Medulloblastoma
B) Craniopharyngioma
Q384 Neurology
A boy with poor linear growth, puberty delay,
worsening headaches, papilledema most likely has:
Medulloblastoma arise from the cerebellar vermis - vomiting, headache, ataxia
A) Medulloblastoma
B) Craniopharyngioma
Craniopharyngioma - a benign tumor derived from remnants of Rathke pouch
(pituitary stalk), found in suprasellar region adjacent to optic chiasm; can result in
compression of pituitary stalk (growth failure due to low TSH or GH, low LH and
FSH lead to puberty delay) and optic chiasm compression (bitemporal
hemianopsia).
Q385 Neurology
A woman with a history of radiation now has
bilateral arm and leg weakness and UMN signs. Vital
signs are normal. MRI shows a homogenously
enhancing mass at C2. What is the diagnosis?

A) Tuberculoma
B) Spinal meningioma
Q385 Neurology
A woman with a history of radiation now has
bilateral arm and leg weakness and UMN signs. Vital
signs are normal. MRI shows a homogenously
enhancing mass at C2. What is the diagnosis?

A) Tuberculoma
B) Spinal meningioma
Q385 Neurology
A woman with a history of radiation now has
bilateral arm and leg weakness and UMN signs. Vital
signs are normal. MRI shows a homogenously
enhancing mass at C2. What is the diagnosis?
(Tuberculoma can compress the spinal cord; but it would also be causing major systemic symptoms)

A) Tuberculoma Exposing ionization is the greatest acquired


B) Spinal meningioma risk factor for spinal meningioma - can lead to
spinal cord compression as seen in this patient.
Q386 Neurology
A 64-year-old man has a mild tremor which
increases during finger-to-nose testing. What is
the treatment?

A) Benzodiazepine
B) Primidone
Q386 Neurology
A 64-year-old man has a mild tremor which
increases during finger-to-nose testing. What is
the treatment?

A) Benzodiazepine
B) Primidone
Q386 Neurology
A 64-year-old man has a mild tremor which
increases during finger-to-nose testing. What is
the treatment?
Benzodiazepines could reduce the tremor, since ET is worsened by

A) Benzodiazepine anxiety; but not a recommended treatment because of dependence


and abuse

B) Primidone (and propranolol)


Q387 Neurology
What is the treatment for the tumor in the image
shown (e.g., causing headaches and weakness)?

A) Chemo
B) Whole brain radiation
C) Surgical removal
Q387 Neurology
What is the treatment for the tumor in the image
shown (e.g., causing headaches and weakness)?

A) Chemo
B) Whole brain radiation
C) Surgical removal
Q387 Neurology
What is the treatment for the tumor in the image
shown (e.g., causing headaches and weakness)?

A) Chemo not chemo (that's for highly malignant brain tumors


B) Whole brain radiation such as glioblastoma and medulloblastoma; and not
whole brain radiation - that's for diffuse metastatic
C) Surgical removal brain disease)
Meningiomas - benign primary brain tumors;
however, they can present with headache, seizure,
and focal neurologic deficits due to mass effect. In
such cases, surgical resection is recommended.
Q388 Neurology
A 63-year-old man has been acutely agitated for 20
minutes after arriving at the post-anesthesia care
unit. He is anxious, confused, and restless. What is
the next step?

A) Lorazepam
B) Reassurance
Q388 Neurology
A 63-year-old man has been acutely agitated for 20
minutes after arriving at the post-anesthesia care
unit. He is anxious, confused, and restless. What is
the next step?

A) Lorazepam
B) Reassurance
Q388 Neurology
A 63-year-old man has been acutely agitated for 20
minutes after arriving at the post-anesthesia care
unit. He is anxious, confused, and restless. What is
the next step?

A) Lorazepam – they’d prolong delirium!


B) Reassurance
Inadequate emergence from general anesthesia (i.e., delayed emergence, emergence
delirium) is typically due to residual effects of anesthetic/adjuvant medication. It is
usually temporary and resolves with reassurance and reorientation
Q389 Neurology
During induction of labor, a woman (with no
significant pregnancy complications) develops
generalized tonic-clonic seizures. What caused
this?

A) Misoprostol
B) Bupivacaine
Q389 Neurology
During induction of labor, a woman (with no
significant pregnancy complications) develops
generalized tonic-clonic seizures. What caused
this?

A) Misoprostol
B) Bupivacaine
Q389 Neurology
During induction of labor, a woman (with no
significant pregnancy complications) develops
generalized tonic-clonic seizures. What caused
this?
Misoprostol can cause abdominal pain and diarrhea, not seizures.
A) Misoprostol
B) Bupivacaine Local anesthetic systemic toxicity is a potential complication of
epidural analgesia; it can cause CNS overactivity (perioral
numbness, metallic taste, tinnitus) and generalized tonic-clonic
seizures.
Q390 Neurology
A baby presents with a rapidly increasing head size,
but she is asymptomatic. What is the next step?

A) Lumbar puncture
B) Reassurance and follow up in a month
C) MRI
Q390 Neurology
A baby presents with a rapidly increasing head size,
but she is asymptomatic. What is the next step?

A) Lumbar puncture
B) Reassurance and follow up in a month
C) MRI
Q390 Neurology
A baby presents with a rapidly increasing head size,
but she is asymptomatic. What is the next step?

A) Lumbar puncture
B) Reassurance and follow up in a month
C) MRI
Hydrocephalus in children is usually due to obstruction (e.g., tumor, infection) that impairs ventricular CSF drainage
and increases intracranial pressure (ICP). In children/adults, who have a fixed cranial vault, increased ICP often results
in headache and vomiting; but babies have open fontanelles that can sometimes accommodate some of the pressure
– so they may be asymptomatic. Examination may show a full fontanelle with widely spaced sutures. Nonspecific
findings can include irritability, poor feeding, and/or developmental delay (e.g., inability to sit with support or roll over
at age 6 months). first step in evaluation of suspected hydrocephalus is neuroimaging. If a baby has an open
fontanelle, ultrafast MRI or ultrasonography of the head can be done.
Q391 Neurology
A 35-year-old woman is brought to the ED due to a
first-time seizure. As per her brother, she has been
acting strangely for several months along with
headaches. Fundoscopy reveals bilateral
papilledema. What is the diagnosis?

A) Frontotemporal lobe atrophy


B) Mass in the frontal lobe
Q391 Neurology
A 35-year-old woman is brought to the ED due to a
first-time seizure. As per her brother, she has been
acting strangely for several months along with
headaches. Fundoscopy reveals bilateral
papilledema. What is the diagnosis?

A) Frontotemporal lobe atrophy


B) Mass in the frontal lobe
Q391 Neurology
A 35-year-old woman is brought to the ED due to a
first-time seizure. As per her brother, she has been
acting strangely for several months along with
headaches. Fundoscopy reveals bilateral
papilledema. What is the diagnosis?

A) Frontotemporal lobe atrophy


B) Mass in the frontal lobe
Tumors can cause elevated ICP leading to papilledema. Frontal lobe involvement can affect
personality, language, motor or executive functions. (Frontotemporal dementia is seen in
patients ages 50-70; doesn't cause headaches or papilledema).
Q392 Neurology
A 60-year-old woman with RA presents with headaches
has bilateral symmetric weakness of the lower limbs
(although arms are fine), UMN signs (hyperreflexia,
upgoing Babinski) and urinary incontinence. What is the
cause?

A) Folate deficiency
B) ALS
C) Cervical myelopathy
D) Parasagittal meningioma
Q392 Neurology
A 60-year-old woman with RA presents with headaches
has bilateral symmetric weakness of the lower limbs
(although arms are fine), UMN signs (hyperreflexia,
upgoing Babinski) and urinary incontinence. What is the
cause?

A) Folate deficiency
B) ALS
C) Cervical myelopathy
D) Parasagittal meningioma
Q392 Neurology
A 60-year-old woman with RA presents with headaches
has bilateral symmetric weakness of the lower limbs
(although arms are fine), UMN signs (hyperreflexia,
upgoing Babinski) and urinary incontinence. What is the
cause?

A) Folate deficiency - doesn't lead to neurologic manifestations; rare due to food fortification; no
headache
B) ALS - mixed UMN and LMN would be present
C) Cervical myelopathy - seen in RA patients, a lesion would cause deficits in the arms as well
D) Parasagittal meningioma - a benign slow growing tumor; can affect just the legs; UMN & LMN
Q393 Neurology
A 5-year-old girl with a ventriculoperitoneal shunt
develops vomiting, irritability, and lethargy, but no fever.
What is the next step?

A) antibiotics
B) lumbar puncture
C) CT scan of head
Q393 Neurology
A 5-year-old girl with a ventriculoperitoneal shunt
develops vomiting, irritability, and lethargy, but no fever.
What is the next step?

A) antibiotics
B) lumbar puncture
C) CT scan of head
Q393 Neurology
A 5-year-old girl with a ventriculoperitoneal shunt
develops vomiting, irritability, and lethargy, but no fever.
What is the next step?

A) antibiotics Shunt malfunction can lead to rapid clinical


decompensation due to recurrence of
B) lumbar puncture hydrocephalus; so we need to assess what the
C) CT scan of head problem is right now - is it the integrity of the
shunt? (no antibiotics because no fever; don't do
LP in suspected hydrocephalus because of risk of
brain herniation)
Q394 Neurology
A 38-year-old woman presents with the "worst headache
of her life"; she also has hypotension, bilateral visual field
defects, and ophthalmoplegia. What is the diagnosis?

A) Subarachnoid hemorrhage
B) Pituitary apoplexy
Q394 Neurology
A 38-year-old woman presents with the "worst headache
of her life"; she also has hypotension, bilateral visual field
defects, and ophthalmoplegia. What is the diagnosis?

A) Subarachnoid hemorrhage
B) Pituitary apoplexy
Q394 Neurology
A 38-year-old woman presents with the "worst headache
of her life"; she also has hypotension, bilateral visual field
defects, and ophthalmoplegia. What is the diagnosis?

A) Subarachnoid hemorrhage
B) Pituitary apoplexy
Pituitary apoplexy is due to hemorrhage or acute ischemia of
the pituitary, usually associated with a large adenoma. Loss of
ACTH can cause central adrenal insufficiency, leading to
hypotension. (In SAH, there is hypertension).
Q395 Neurology
A 23-year-old male with HIV with a CD4<30 has a
seizure. MRI shows a solitary irregular ring-
enhancing lesion in the periventricular area. CSF is
positive for EBV. What is the diagnosis?

A) Toxoplasmosis
B) Primary CNS lymphoma
Q395 Neurology
A 23-year-old male with HIV with a CD4<30 has a
seizure. MRI shows a solitary irregular ring-
enhancing lesion in the periventricular area. CSF is
positive for EBV. What is the diagnosis?

A) Toxoplasmosis
B) Primary CNS lymphoma
Q395 Neurology
A 23-year-old male with HIV with a CD4<30 has a
seizure. MRI shows a solitary irregular ring-
enhancing lesion in the periventricular area. CSF is
positive for EBV. What is the diagnosis?

A) Toxoplasmosis
B) Primary CNS lymphoma
Cerebral toxoplasmosis is common in patients with advanced AIDS (CD4
count <50/mm3) but multiple ring-enhancing lesions are usually present,
and EBV DNA is not seen in the CSF
Q396 Neurology
A 4-month-old boy has a head circumference at
96th percentile but no symptoms, and this has
been consistent. What is the next step?

A) MRI
B) Reassurance & observation
Q396 Neurology
A 4-month-old boy has a head circumference at
96th percentile but no symptoms, and this has
been consistent. What is the next step?

A) MRI
B) Reassurance & observation
Q396 Neurology
A 4-month-old boy has a head circumference at
96th percentile but no symptoms, and this has
been consistent. What is the next step?

A) MRI
B) Reassurance & observation
Macrocephaly is a head circumference >97th percentile. The
condition is most likely benign (familial) in a patient with normal
development and normal examination; management is
reassurance and observation.
Q397 Neurology
A 55-year-old man has repeated episodes of
violent episodes at night; his dreams lack muscle
atonia. He will most likely develop:

A) an alpha-synuclein neurodegenerative disorder


B) obstructive sleep apnea
Q397 Neurology
A 55-year-old man has repeated episodes of
violent episodes at night; his dreams lack muscle
atonia. He will most likely develop:

A) an alpha-synuclein neurodegenerative disorder


B) obstructive sleep apnea
Q397 Neurology
A 55-year-old man has repeated episodes of
violent episodes at night; his dreams lack muscle
atonia. He will most likely develop:

A) an alpha-synuclein neurodegenerative disorder


B) obstructive sleep apnea sleep behavior disorder eventually develop
Up to 90% of patients with idiopathic REM

either PD, LBD, or another disorder of


alpha-synuclein neurodegeneration
(multiple system atrophy) - it's considered a
prodrome! (NO association with sleep
apnea)
Q398 Neurology
A woman has recurrent forceful contraction of
the eyelids, triggered by bright lights. Sometimes
when she touches the skin around the eye, it ends
the episode. What is the diagnosis?

A) Uveitis
B) Blepharospasm
Q398 Neurology
A woman has recurrent forceful contraction of
the eyelids, triggered by bright lights. Sometimes
when she touches the skin around the eye, it ends
the episode. What is the diagnosis?

A) Uveitis
B) Blepharospasm
Q398 Neurology
A woman has recurrent forceful contraction of
the eyelids, triggered by bright lights. Sometimes
when she touches the skin around the eye, it ends
the episode. What is the diagnosis?

A) Uveitis Blepharospasm (a form of focal dystonia) can be caused by


B) Blepharospasm movement disorders (PD) or medications (e.g., antipsychotics)
- but usually idiopathic. (When there's also jaw spasm, it's
called Meige syndrome). Tx: trigger avoidance (e.g.,
sunglasses); botulinum toxin if severe symptoms
Q399 Neurology
A patient has excessive daytime sleepiness,
hallucinations on falling asleep, and sleep
paralysis. What is first-line treatment (besides
behavior modification)?

A) clonazepam
B) modafinil
Q399 Neurology
A patient has excessive daytime sleepiness,
hallucinations on falling asleep, and sleep
paralysis. What is first-line treatment (besides
behavior modification)?

A) clonazepam
B) modafinil
Q399 Neurology
A patient has excessive daytime sleepiness,
hallucinations on falling asleep, and sleep
paralysis. What is first-line treatment (besides
behavior modification)?

A) clonazepam
B) modafinil First-line for narcolepsy; stimulant;
sometimes used in sleep apnea
Q400 Neurology
The most common central nervous tumor in
children is:

A) Astrocytoma
B) Neuroblastoma
C) Craniopharyngioma
Q400 Neurology
The most common central nervous tumor in
children is:

A) Astrocytoma
B) Neuroblastoma
C) Craniopharyngioma
Q400 Neurology
The most common central nervous tumor in
children is:

A) Astrocytoma Usually occurs in the frontal, parietal, or temporal lobe


B) Neuroblastoma Abdominal mass; end-stage can metastasize to the brain; rare
C) Craniopharyngioma Arise in the sella tursica
Intro Neurology
Updates in Neurology
Intro Neurology
Updates in Neurology

• tPA (e.g., alteplase) can be given within 4.5 hours after onset of symptoms
(used to be 3 hours)
Intro Neurology
Updates in Neurology

• tPA (e.g., alteplase) can be given within 4.5 hours after onset of symptoms
(used to be 3 hours)
• Restless leg syndrome first-line treatment is Gabalin or Pregabalin (used to
be dopamine agonists)
Intro Neurology
Updates in Neurology

• tPA (e.g., alteplase) can be given within 4.5 hours after onset of symptoms
(used to be 3 hours)
• Restless leg syndrome first-line treatment is Gabalin or Pregabalin (used to
be dopamine agonists)
• Edaravone was approved in 2017 for ALS (given with Riluzole)
Intro Neurology
Updates in Neurology

• tPA (e.g., alteplase) can be given within 4.5 hours after onset of symptoms
(used to be 3 hours)
• Restless leg syndrome first-line treatment is Gabalin or Pregabalin (used to
be dopamine agonists)
• Edaravone was approved in 2017 for ALS (given with Riluzole)
• India Ink for cryptococcus was discontinued in 2019; now it’s antigen testing
Intro Neurology
Updates in Neurology

• tPA (e.g., alteplase) can be given within 4.5 hours after onset of symptoms
(used to be 3 hours)
• Restless leg syndrome first-line treatment is Gabalin or Pregabalin (used to
be dopamine agonists)
• Edaravone was approved in 2017 for ALS (given with Riluzole)
• India Ink for cryptococcus was discontinued in 2019; now it’s antigen testing
• Shingrix is given at age 50 (used to be 60)
Intro Neurology
Updates in Neurology

• tPA (e.g., alteplase) can be given within 4.5 hours after onset of symptoms
(used to be 3 hours)
• Restless leg syndrome first-line treatment is Gabalin or Pregabalin (used to
be dopamine agonists)
• Edaravone was approved in 2017 for ALS (given with Riluzole)
• India Ink for cryptococcus was discontinued in 2019; now it’s antigen testing
• Shingrix is given at age 50 (used to be 60)
• VMAT2 inhibitors (tetrabenazine) are increasingly preferred for Tourette's as
initial treatment (over antipsychotics); Guanfacine is great if patient also has
ADHD
Intro Neurology
Updates in Neurology

• tPA (e.g., alteplase) can be given within 4.5 hours after onset of symptoms
(used to be 3 hours)
• Restless leg syndrome first-line treatment is Gabalin or Pregabalin (used to
be dopamine agonists)
• Edaravone was approved in 2017 for ALS (given with Riluzole)
• India Ink for cryptococcus was discontinued in 2019; now it’s antigen testing
• Shingrix is given at age 50 (used to be 60)
• VMAT2 inhibitors (tetrabenazine) are increasingly preferred for Tourette's as
initial treatment (over antipsychotics); Guanfacine is great if patient also has
ADHD
• For CJD, CSF real-time quaking-induced conversion (RT-QULC)
replaced14-3-3 protein detection in the CSF since it’s more sensitive and
specific

You might also like